Você está na página 1de 101

Environment-Ecology-Biodiversity- Subject wise compilation of 60 Day Plan- 2016

Q.1) Consider the following about Global Environment Facility (GEF) .It serves as financial
mechanism for?
1. Convention on Biological Diversity (CBD)
2. United Nations Framework Convention on Climate Change (UNFCCC)
3. Stockholm Convention on Persistent Organic Pollutants (POPs)
4. Montreal Protocol
5. UN Convention to Combat Desertification (UNCCD)
6. Minamata Convention on Mercury
Select the correct code
a) 2, 3, 4, 5 and 6
b) 2, 4 and 5
c) 1, 2, 3, 4 and 5
d) 1, 2, 3, 5 and 6
Q.1) Solution (d)
The GEF serves as financial mechanism for the following conventions:
Convention on Biological Diversity (CBD)
United Nations Framework Convention on Climate Change (UNFCCC)
Stockholm Convention on Persistent Organic Pollutants (POPs)
UN Convention to Combat Desertification (UNCCD)
Minamata Convention on Mercury
Source- Picked from India Year Book, Chapter on Environment (Global Environment Facility)
Q.2) Consider the following
1. International Day for Biological Diversity is celebrated on 29th December
2. It is held on December 29 to celebrate the day the Convention on Biological Diversity
went into effect.
Select the correct code
a) Only 1
b) Only 2
c) Both
d) None
Q.2) Solution (d)
It is now celebrated on 22nd May. Earlier it was celebrated on 29th December until 2000.
Since then it is celebrated on 22nd May to commemorate the adoption of the Convention
on May 22, 1992 at the Rio Earth Summit.

www.iasbaba.com

Environment-Ecology-Biodiversity- Subject wise compilation of 60 Day Plan- 2016


http://www.business-standard.com/article/government-press-release/biodiversity-forsustainable-development-environment-minister-s-message-on-international-day115052200177_1.html
Q.3) Consider the following
1. Nagoya Protocol came into force before Cartagena Protocol
2. Nagoya Protocol comes under Convention on Biological Diversity
3. The Nagoya Protocol on Biosafety is an international agreement on biosafety as a
supplement to the Convention on Biological Diversity
Select the correct code
a) Only 1
b) Only 2
c) 1 and 3
d) 1, 2 and 3
Q.3) Solution (b)
Source- Picked from India Year Book, Chapter on Environment (CBD)
Nagoya Protocol came into force in 2014 while Cartagena in 2003
Q.4) National Biodiversity Authority (NBA) is
1. An autonomous organization
2. A statutory organization
3. A regulatory organization
Select the correct code
a) 1 and 2
b) Only 2
c) 2 and 3
d) 1, 2 and 3
Q.4) Solution (d)
The Biological Diversity Act, 2002 came into force in 2003. The Act extents to the whole of
India. The objectives of the Act are conservation, sustainable utilization and fair and
equitable sharing of benefits arising out of the use of biological resources and associated
knowledge. The Act is being implemented in a three-tiered institutional structure. The NBA
is a body corporate established in accordance with the provisions of Section 8 of the
Biological Diversity Act, 2002, at Chennai in 2003. It is an autonomous, statutory and
regulatory organization which is intended to implement the provisions of Biological Diversity
Act, 2002.
Source- Picked from India Year Book, Chapter on Environment (NBA)

www.iasbaba.com

Environment-Ecology-Biodiversity- Subject wise compilation of 60 Day Plan- 2016

Q.5) Consider the following statements regarding Mangroves


1. India accounts for nearly 10% of worlds mangrove vegetation
2. There has been a net decreases in the mangrove cover of India over last few years
3. Andhra Pradesh has more mangrove cover than Gujarat
Select the correct code
a) Only 1
b) 1 and 2
c) 1, 2 and 3
d) None
Q.5) Solution (d)
India accounts for nearly 3 % of worlds mangrove vegetation
There has been a net increase in the mangrove cover of India over last few years (latest
assessment in 2015 has more mangrove cover than assessment of 2013)
Andhra Pradesh (367 km2) has lesser mangrove cover than Gujarat ( 1107 km2)
Source- Picked from India Year Book, Chapter on Environment (Mangroves) and Indian State
of Forest Report, 2015
http://fsi.nic.in/isfr-2015/isfr-2015-mangrove-cover.pdf
Q.6) According to Ramsar Convention definition of Wetland includes
1. Area that is saturated with water permanently
2. Areas of marine water the depth of which at low tide does not exceed eight metres
Select the correct code
a) Only 1
b) Only 2
c) Both
d) None
Q.6) Solution (a)
Under the Ramsar international wetland conservation treaty, wetlands are defined as
follows:
"...wetlands are areas of marsh, fen, peatland or water, whether natural or artificial,
permanent or temporary, with water that is static or flowing, fresh, brackish or salt,
including areas of marine water the depth of which at low tide does not exceed six metres."
"[Wetlands] may incorporate riparian and coastal zones adjacent to the wetlands, and
islands or bodies of marine water deeper than six metres at low tide lying within the
wetlands."

www.iasbaba.com

Environment-Ecology-Biodiversity- Subject wise compilation of 60 Day Plan- 2016


Q.7) Consider the following w.r.t Ecotones and Ecoclines
1. an ecocline describes a variation in species prevalence and is often not strictly
dependent on a major physical factor separating an ecosystem from another, with
resulting habitat variability.
2. an ecotone is a variation of the physicochemical environment dependent of one or two
physico-chemical factors of life, and thus presence/absence of certain species.
Select the correct code
a) Only 1
b) Only 2
c) Both
d) None
Q.7) Solution (d)
Definition is interchanged.
Q.8) Consider the following regarding Keystone Species
1. Keystone species have lower levels of biomass in the trophic pyramid relative to the
importance of their role.
2. Sea Otters are example of keystone species
Select the correct code
a) Only 1
b) Only 2
c) Both
d) None
Q.8) Solution (c)
http://www.thehindubusinessline.com/blink/explore/age-of-extinction/article7433560.ece
https://en.wikipedia.org/wiki/Keystone_species#Examples
Q.9) Consider the following regarding Fly Ash
1. It is used as a fertilizer
2. It can be used in soil stabilization
3. It is used as a replacement for Portland cement in concrete.
Select the correct code
a) 2 and 3
b) 1 and 2
c) Only 3
d) 1, 2 and 3

www.iasbaba.com

Environment-Ecology-Biodiversity- Subject wise compilation of 60 Day Plan- 2016

Q.9) Solution (d)


All are correct
Source- Reference picked from India Year Book- Environment (Fly Ash Utilization)
Q.10) Electronic waste or e-waste describes discarded electrical or electronic devices.
Consider the following regarding E- waste
1. Beryllium
2. Cadmium
3. Gold
4. Lithium
5. Uranium
6. Silver
Which of the above are found in e-waste?
a) 1, 2, 3, 4 and 5
b) 1, 2, 3, 4 and 6
c) 1, 2, 3 and 4
d) 1, 2 and 4
Q.10) Solution (b)
Except Uranium which is radioactive, all of the above are found in e-waste.
The Ministry of Environment, Forest and Climate Change has recently notified the E-Waste
Management Rules, 2016 in supersession of the e-waste (Management & Handling) Rules,
2011.
http://pib.nic.in/newsite/PrintRelease.aspx?relid=138319
http://www.fastcoexist.com/3050542/the-gold-and-silver-and-more-in-our-e-waste-isworth-billions-of-dollars
http://www.thehindu.com/news/national/other-states/ewaste-can-produce-more-goldand-silver-than-mining/article8104059.ece
Q.11) Consider the following statements:
1. Eurythermal organisms can tolerate and thrive in wide range of temperature.
2. Warm blooded animals can maintain a constant body temperature even when the
surrounding temperature is changing.
3. Stenothermal organisms are restricted to a small range of temperature.
4. Cold blooded animals generally hibernate to protect themselves from extreme climatic
conditions.
Which of the above statements are correct:
a) 1 and 3

www.iasbaba.com

Environment-Ecology-Biodiversity- Subject wise compilation of 60 Day Plan- 2016


b) 2 and 4
c) All of the above
d) None of the above
Q.11) Solution (c)
Self explanatory. (Class XII NCERT)
Q.12) In Equatorial rainforest, the undergrowth on the floor is very less. The correct
reason behind this is:
a) Very less sunlight reaches the ground for survival of plants due to thick canopies.
b) The hardwood trees excrete toxic resins which do not allow the plants to grow.
c) Due to high population of herbivorous species.
d) The soil of rainforest is less fertile due to heavy leaching of minerals.
Q.12) Solution (a)
Equatorial rain forests have thick canopy cover which does not allow the sunlight to reach
the floor. There is stiff competition to between the trees to catch the maximum sunlight.
Due to this there is very little undergrowth.
Q.13) Which of the following statements are correct about life processes of plants.
1. The process of making of food with the help of photosynthesis takes place only during
the day.
2. The process of respiration takes place during day and night both.
Which of the above statements are incorrect:
a) 1 only
b) 2 only
c) Both 1 and 2
d) Neither 1 nor 2
Q.13) Solution (a)
The plants prepare their own food by the process of photosynthesis. The process has two
phases, the light reaction and the dark reaction. The light reaction takes place in the
presence of sunlight, while the dark reaction does not require light and continues both
during day and night. The byproduct of photosynthesis is Oxygen gas.
The plants burn their food to produce energy for their survival. This process is respiration.
The plants take in O2 and release CO2. Since light reaction does not take place during night,
the respiration dominates. Thats why it is not wise to sleep under a tree at night as it
releases a lot of Carbon dioxide.
Q.14) Consider the following statements:

www.iasbaba.com

Environment-Ecology-Biodiversity- Subject wise compilation of 60 Day Plan- 2016


1. Despite various external conditions, the organisms maintain constant internal
environment like temperature and osmotic pressure. This process is called Homeostasis.
2. During winters the human body shivers to generate heat and maintain the body
temperature of 37oC.
3. During summers the body sweats profusely to maintain the temperature.
4. Plants also have the same mechanism as animals to maintain homeostasis.
Which of the above statements are correct:
a) 1 only
b) 1,2 and 3
c) 1, 3 and 4
d) All of the above
Q.14) Solution (b)
The organism should try to maintain the constancy of its internal environment (a process
called homeostasis) despite varying external environmental conditions that tend to upset its
homeostasis.
Some organisms are able to maintain homeostasis by physiological (sometimes behavioural
also) means which ensures constant body temperature, constant osmotic concentration,
etc. All birds and mammals, and a very few lower vertebrate and invertebrate species are
indeed capable of such regulation (thermoregulation and osmoregulation). Evolutionary
biologists believe that the success of mammals is largely due to their ability to maintain a
constant body temperature and thrive whether they live in Antarctica or in the Sahara
desert.
The mechanisms used by most mammals to regulate their body temperature are similar to
the ones that we humans use. We maintain a constant body temperature of 370C. In
summer, when outside temperature is more than our body temperature, we sweat
profusely. The resulting evaporative cooling, similar to what happens with a desert cooler in
operation, brings down the body temperature. In winter when the temperature is much
lower than 370C, we start to shiver, a kind of exercise which produces heat and raises the
body temperature.
Plants, on the other hand, do not have such mechanisms to maintain internal temperatures.
Q.15) Which of the following statements are correct about Keoladeo National Park.
1. It is a man-made and man-managed wetland.
2. It is famous for migratory birds which come to visit the park during winters.
3. It is a World Heritage Site.
Select the code from the following:
a) 1 and 2

www.iasbaba.com

Environment-Ecology-Biodiversity- Subject wise compilation of 60 Day Plan- 2016


b) 2 and 3
c) 1 and 3
d) All of the above
Q.15) Solution (d)
Keoladeo Ghana National Park formerly known as the Bharatpur Bird Sanctuary in
Bharatpur, Rajasthan, India is a famous avifauna sanctuary that hosts thousands of birds,
especially during the winter season. It is a man-made and man-managed wetland and also a
World Heritage Site.
Q.16) Consider the following statements:
1. The exchange of air (breathing) in plants takes place only through stomata present in the
leaves.
2. Many desert plants have their stomata in deep pits to minimize water loss by
evaporation.
3. Desert plants have a mechanism that their stomata remain closed during day time.
Which of the above statements are incorrect:
a) 1 and 2
b) 2 and 3
c) 1 and 3
d) None of the above
Q.16) Solution (a)
Apart from stomata plants and trees also breathe through lenticles, which are small pores
present on their stem.
Many desert plants have their stomata in deep pits to minimize water loss by transpiration
and not evaporation.
Q.17) At higher elevations, people generally have breathing problems due to low pressure
and lack of oxygen. This is called altitude sickness. Soon the body acclimatise this situation
and and breathing becomes normal. Which of the following functions does the body take
to adapt to this situation:
1. The body increases red blood cell production.
2. Body decreases the binding capacity of hemoglobin.
3. Body increases the breathing rate.
Select the correct code from the following:
a) 1 only
b) 2 and 3
c) 1 and 3

www.iasbaba.com

Environment-Ecology-Biodiversity- Subject wise compilation of 60 Day Plan- 2016


d) All of the above
Q.17) Solution (d)
At higher altitude people experience altitude sickness. Its symptoms include nausea, fatigue
and heart palpitations. This is because of low atmospheric pressure, the body doesnt get
enough oxygen. But gradually one gets acclimatized as body compensates low oxygen
availability by producing more red blood cells, decreasing the binding capacity of
hemoglobin and by increasing breathing rate. (NCERT XII Biology)
Q.18) Consider the following statements:
1. Detrivores like earthworm, breakdown detritus into smaller particles. This process is
called fragmentation.
2. Bacterial and fungal enzymes degrade detritus into simpler inorganic substances. This
process is called as catabolism.
Which of the above statements are correct:
a) 1 only
b) 2 only
c) Both 1 and 2
d) Neither 1 nor 2
Q.18) Solution (c)
Detritivores (e.g., earthworm) break down detritus into smaller particles. This process is
called fragmentation. By the process of leaching, watersoluble inorganic nutrients go down
into the soil horizon and get precipitated as unavailable salts. Bacterial and fungal enzymes
degrade detritus into simpler inorganic substances. This process is called as catabolism.
Q.19) Consider the following statements regarding Humus:
1. Humus is a finely divided partially decomposed organic material in soil, whose origin
cannot be traced by looking at it.
2. Humus is colloidal and helps in binding minerals in the top soil and making them
available to the roots of trees.
3. It is highly resistant to microbial action and decomposes at a very slow rate.
Which of the above statements are correct:
a) 1 and 2
b) 2 and 3
c) 1 and 3
d) All of the above
Q.19) Solution (d)

www.iasbaba.com

Environment-Ecology-Biodiversity- Subject wise compilation of 60 Day Plan- 2016


Humification leads to accumulation of a dark coloured amorphous substance called humus
that is highly resistant to microbial action and undergoes decomposition at an extremely
slow rate. Being colloidal in nature it serves as a reservoir of nutrients. The humus is further
degraded by some microbes and release of inorganic nutrients occur by the process known
as mineralisation.
Q.20) Consider the following statements regarding energy flow from one tropic level to
other tropic level:
1. The energy flow is always unidirectional, i.e. from lower tropic level to higher tropic
level.
2. The energy transfer in tropic levels follows a 10% law, i.e. only 10 percent energy is
transferred from lower tropic level to the next tropic level.
Which of the above statements are correct:
a) 1 only
b) 2 only
c) Both 1 and 2
d) Neither 1 nor 2
Q.20) Solution (c)
Refer Biology Class XII NCERT, chapter 14
Q.21) Which of the following is/are legally binding?
1. Nagoya Protocol
2. Convention on Biological Diversity
3. Paris Agreement on Climate Change
Select the correct code
a) Only 2
b) 1 and 2
c) 1, 2 and 3
d) 1 and 3
Q.21) Solution (c)
http://cdkn.org/2015/12/feature-is-the-paris-agreement-legally-binding/?loclang=en_gb
Doubt: please verify question 1...Whether Paris Agreement is legally binding is debatable..
- Dilutes the principles of equity adopted by the Kyoto protocol in 1997
- Does away with the legally binding emission targets for the developed countries that was
agreed upon in Kyoto

www.iasbaba.com

10

Environment-Ecology-Biodiversity- Subject wise compilation of 60 Day Plan- 2016


- Does not enforce any legally binding commitments from developed countries in the future
as countries will set their own targets and no penalties will be imposed for failing to meet
those - Developed nations cannot be filed for liability or compensation for loss and damage
incurred by poorer nations as a result of historical emissions
Clarification
There are various interpretations but logically it legally non-binding. Legally binding will
always have some penalty or actions in return.
http://thinkprogress.org/climate/2015/12/14/3731715/paris-agreement-is-an-actualagreement/
So answer should be (b)
Q.22) Which among the following can effectively be examples of Secondary Succession?
1. Formation of a new Island after volcanic eruption
2. The renewal of a forest after a fire
3. A flooded land
4. The renewal of a crop after harvesting
Select the correct option
a) 1, 2 and 3
b) 2, 3 and 4
c) 1, 3 and 4
d) All
Q.22) Solution (b)
Examples of secondary succession include:
The renewal of a forest after a fire: The fire itself destroys a majority of different types
of trees and plant life. Because seeds and roots and other plant and tree parts remain in
and on the soil, gradually the plants and trees begin to grow again and eventually return
to the state of the original ecosystem.
The renewal of a crop after harvesting: A crop is completed harvested when it becomes
ripe. Without new seeds being planted, the crop can regenerate the following year due
to the plants and seeds that remained after harvesting.
A forest renews after logging: A large amount of trees were chopped down by loggers in
order to create building materials. Over time, trees grow in and the area returns to its
previous state.
Renewal after disease: A plant population can be very negatively affected by a variety of
infectious plant diseases. If the entire population dies, but the soil and roots remain, it is
possible for secondary succession to occur and for the population of those plants to
return.

www.iasbaba.com

11

Environment-Ecology-Biodiversity- Subject wise compilation of 60 Day Plan- 2016

A flood can ruin farmlands. However, because the soil remains after the waters recede,
over the course of many years a natural secondary succession can occur and the
vegetation that had previously grown there can grow again.
Plants can be very susceptible to attack from pests, particularly if there is an
overpopulation of those pests. When this occurs, the plant population in one area can
be completely destroyed. However, when the pest overpopulation is resolved, the plants
are able to live again and thrive in the soil in which they previously had lived.

Q. 23) Consider the following with regard to National Green Tribunal (NGT)
1. NGT is bound by procedure laid down under the Code of Civil Procedure, 1908
2. The tribunal has Original as well as Appellate jurisdiction
3. Tribunal is competent to hear cases for several acts such as Forest Right Act, Biological
Diversity Act, Environment (Protection) Act, Water & Air (Prevention & control of
Pollution) Acts.
Select the incorrect code
a) 1 and 2
b) Only 2
c) 1 and 3
d) 1, 2 and 3
Q.23) Solution (c)
The Tribunal's dedicated jurisdiction in environmental matters shall provide speedy
environmental justice and help reduce the burden of litigation in the higher courts. The
Tribunal shall not be bound by the procedure laid down under the Code of Civil Procedure,
1908, but shall be guided by principles of natural justice. The Tribunal is mandated to make
and endeavour for disposal of applications or appeals finally within 6 months of filing of the
same.
The Tribunal has Original Jurisdiction on matters of substantial question relating to
environment (i.e. a community at large is affected, damage to public health at broader
level) & damage to environment due to specific activity (such as pollution).
Also Tribunal is competent to hear cases for several acts such as Forest (Conservation) Act,
Biological Diversity Act, Environment (Protection) Act, Water & Air (Prevention & control of
Pollution) Acts etc. and also have appellate jurisdiction related to above acts after
establishment of Tribunal within a period of 30 days of award or order received by
aggrieved party.
Q.24) Consider the following statements about the Green India Mission (GIM). Its
mission is/are
1. To increase forest/tree cover to the extent of 5 million hectares
2. To enhance ecosystem services like timber and non-timber produce

www.iasbaba.com

12

Environment-Ecology-Biodiversity- Subject wise compilation of 60 Day Plan- 2016


3. To enhance provisioning services like carbon sequestration, hydrological services and
biodiversity
Identify the incorrect statement/s
a) 1 and 2
b) 2 and 3
c) Only 1
d) 1 and 3
Q.24) Solution (b)
National Mission for a Green India (GIM)
To increase forest/tree cover to the extent of 5 million hectares (mha) and improve quality
of forest/tree cover on another 5 mha of forest/non-forest lands;
To improve/enhance eco-system services like carbon sequestration and storage (in forests
and other ecosystems), hydrological services and biodiversity; along with provisioning
services like fuel, fodder, and timber and non-timber forest produces (NTFPs); and to
increase forest based livelihood income of about 3 million households.
Q.25) Loss of biodiversity in a region may lead to
1. Decline in plant production
2. Increased resistance to environmental perturbations such as drought
3. Decreased variability in certain ecosystem processes such as plant productivity, water
use, and pest and disease cycles
Select the correct statement/s
a) 1 and 2
b) Only 3
c) Only 1
d) Only 2
Q.25) Solution (c)
Lowered resistance to environmental perturbations such as drought and increased
variability in certain ecosystem processes such as plant productivity, water use, and pest
and disease cycles
Q.26) Consider the following assumptions regarding Pugmark method
1. The entire potential tiger habitat in India had to be effectively covered during the
pugmark census.
2. All the four paw prints of every individual tiger in the surveyed area had to be detected
during the censuses.

www.iasbaba.com

13

Environment-Ecology-Biodiversity- Subject wise compilation of 60 Day Plan- 2016


3. The same hind pugmark of each one of these individual tigers must be lifted from
suitable and comparable substrates or from standardized soil track-plots.
4. The shape of each pugmark lifted had to be recorded without distortion by the
thousands of census personnel involved in the operation.
5. Supervisory officials were expected to be subsequently able to segregate the pugmarks
of each individual tiger correctly, based on footprint shape, track measurements, and
prior local knowledge.
Select the appropriate code
a) Failure of assumptions 1 and 2 would lead to under-counts
b) Failure of assumptions 3 and 4 would lead to over-counts
c) Failure of assumption 5 could lead to either undercounts or over-counts.
d) a, b and c holds true
Q.26) Solution (d)
Learn about Pugmark technique.
http://www.conservationindia.org/articles/why-the-pugmark-census-used-to-monitor-tigerpopulations-failed
Q.27) Consider the following about South Asia Wildlife Enforcement Network
1. India is a formal member of SAWEN
2. SAWEN, South Asia Wildlife Enforcement Network is an initiative to control the
transboundary Wildlife Crime
Select the correct statement/s
a) Only 1
b) Only 2
c) Both
d) None
Q.27) Solution (c)
http://pib.nic.in/newsite/PrintRelease.aspx?relid=138834
Q.28) Consider the following regarding Bharat Stage (BS) Emission Norms in India
1. Currently, India is following BS IV Norms
2. India has planned to implement BS V norms from 2019
3. BS VI, particulate matter emission for diesel cars and nitrogen oxide levels are expected
to be substantially higher than in BS IV.
Select the incorrect code
a) 1 and 2

www.iasbaba.com

14

Environment-Ecology-Biodiversity- Subject wise compilation of 60 Day Plan- 2016


b) Only 2
c) 2 and 3
d) Only 3
Q.28) Solution (c)
India has planned to switch directly from BS IV to BS VI, skipping BS V
http://www.thehindubusinessline.com/opinion/all-you-wanted-to-know-about-bs-viemission-norms/article8120879.ece
Q.29) The Red Data List published by the International Union for Conservation of Nature
and Natural Resources (IUCN) contain lists of
1. Information on plants, fungi and animals
2. Information on plants and animals only
3. It divides species into Six categories
Select the correct answer using the codes given below
a) 1 and 3
b) 1, 2 and 3
c) Only 3
d) Only 1
Q.29) Solution (d)
http://www.iucnredlist.org/about/introduction
Q.30) Sacred groves are, tracts of forest were set aside, and all the trees and wildlife
within were venerated and given total protection. Sacred groves are found in following
places
1. Melghat Maharashtra
2. Aravalli Rajasthan
3. Western ghats - Karnataka and Maharashtra
4. Sarguja, Chanda and Bastar Madhya Pradesh
Select the correct answers
a) 1, 3 and 4 only
b) 2, 3 and 4 only
c) None of the above
d) All of the above
Q.30) Solution (b)
India has also a history of religious and cultural traditions that emphasised protection of
nature. In many cultures, tracts of forest were set aside, and all the trees and wildlife within

www.iasbaba.com

15

Environment-Ecology-Biodiversity- Subject wise compilation of 60 Day Plan- 2016


were venerated and given total protection. Such sacred groves are found in Khasi and Jaintia
Hills in Meghalaya, Aravalli Hills of Rajasthan, Western Ghat regions of Karnataka and
Maharashtra and the Sarguja, Chanda and Bastar areas of Madhya Pradesh. In Meghalaya,
the sacred groves are the last refuges for a large number of rare and threatened plants.
Doubt- Bastar Sacred Groove known as 'Deogudi' is not in MP but in Chattishgarh. Hence
answer must be (c)
Clarification: There is no denying
time of Harappan Civilization :P

How the hell we forgot that NCERTs were written at the

So answer is (c) in this case.


Q.31) Which of the floral groups are not found in India?
1. Algae
2. Fungi
3. Bacteria
4. Lichens
5. Bryophytes
6. Pteridophytes
7. Gymnosperms
8. Angiosperms
Select the correct answers
a) 6 only
b) 6, 7 and 8 only
c) 5, 6, and 8
d) All of the above are found in India
Q.31) Solution (d)
All the floral groups found in India. Simple examples can be read from class 9 ncert
Q.32) What is common to the techniques (i) in vitro fertilisation, (ii) Cryo preservation and
(iii) tissue culture?
a) All are in situ conservation methods.
b) All are ex situ conservation methods.
c) All require ultra-modern equipment and large space.
d) All are methods of conservation of extinct organisms.
Q.32) Solution (b)

www.iasbaba.com

16

Environment-Ecology-Biodiversity- Subject wise compilation of 60 Day Plan- 2016


Q.33) Consider the following statements about Biodiversity
1. In biosphere diversity exists only at species level, genes level and ecological level.
2. It is the term popularized by socio biologist Robert May.
3. The Western Ghats have a greater amphibian species diversity than the Eastern Ghats
Select the incorrect answer:
a) 1 only
b) Only 1 and 2
c) Only 1, 2 and 3
d) All of the above
Q.33) Solution (b)
In our biosphere immense diversity (or heterogeneity) exists not only at the species level
but at all levels of biological organisation ranging from macromolecules within cells to
biomes. Biodiversity is the term popularised by the socio biologist Edward Wilson to
describe the combined diversity at all the levels of biological organisation.
The most important of them are
Genetic diversity: A single species might show high diversity at the genetic level over its
distributional range. The genetic variation shown by the medicinal plant Rauwolfia vomitoria
growing in different Himalayan ranges might be in terms of the potency and concentration
of the active chemical (reserpine) that the plant produces. India has more than 50,000
genetically different strains of rice, and 1,000 varieties of mango.
Species diversity: The diversity at the species level. For example, the Western Ghats have
greater amphibian species diversity than the Eastern Ghats.
Ecological diversity: At the ecosystem level, India, for instance, with its deserts, rain forests,
mangroves, coral reefs, wetlands, estuaries, and alpine meadows has greater ecosystem
diversity than a Scandinavian country like Norway.
Source 12th Biology NCERT chapter biodiversity and conservation P.no. 258
Q.34) Consider following statements about IUCN RED LIST
1. It uses a set of criteria which are not relevant to all species and all regions of the world.
2. The IUCN Red List is recognized as the most authoritative guide to the status of
biological diversity.
Select the correct answers
a) Only 1
b) Only 2
c) Both
d) None
Q.34) Solution (b)

www.iasbaba.com

17

Environment-Ecology-Biodiversity- Subject wise compilation of 60 Day Plan- 2016


http://www.iucn.org/resources/conservation-tools/iucn-red-list-threatened-species
Q.35) Consider the following statements
1. Agasthyamala Biosphere Reserve has been included in UNESCOs list of World Biosphere
Reserve Network.
2. Agasthyamala Biosphere Reserve rests in between Tamil Nadu and Karnataka
3. Kannikarans are the indigenous tribes residing in this area
Select the incorrect answer
a) 2 only
b) 1 and 3 only
c) 2 and 3 only
d) None of the above
Q.35) Solution (a)
http://www.thebetterindia.com/49714/agasthyamala-unesco-biosphere-reserve-network/
Q.36) The ecological footprint is a measure of human demand on the Earth's ecosystems.
It measures1. How much of the biological capacity of the planet is demanded by a given human
activity or population
2. Ecological Footprints is calculated for overall activity of a nation or population and
activities such as industrialization etc.
3. The disadvantage of Ecological Footprint is that it cannot be calculated for individual
people or area.
Select the incorrect statement/s
a) 1 and 2
b) Only 2
c) 1 and 3
d) Only 3
Q.36) Solution (d)
The Ecological Footprint is a resource accounting tool that measures how much biologically
productive land and sea is used by a given population or activity, and compares this to how
much land and sea is available. Productive land and sea areas support human demands for
food, fibre, timber, energy, and space for infrastructure. These areas also absorb the waste
products from the human economy. The Ecological Footprint measures the sum of these
areas, wherever they physically occur on the planet. The Ecological Footprint is used widely
as a management and communication tool by governments, businesses, educational
institutions, and non-governmental organizations.

www.iasbaba.com

18

Environment-Ecology-Biodiversity- Subject wise compilation of 60 Day Plan- 2016

Ecological Footprint accounts answer a specific research question: how much of the
biological capacity of the planet is demanded by a given human activity or population? To
answer this question, the Ecological Footprint measures the amount of biologically
productive land and water area an individual, a city, a country, a region, or all of humanity
uses to produce the resources it consumes and to absorb the waste it generates with
todays technology and resource management practices. This demand on the biosphere can
be compared to biocapacity, a measure of the amount of biologically productive land and
water available for human use. Biologically productive land includes areas such as cropland,
forest, and fishing grounds, and excludes deserts, glaciers, and the open ocean
Ecological Footprints can be calculated for individual people, groups of people (such as a
nation), and activities (such as manufacturing a product).
The Ecological Footprint of a person is calculated by considering all of the biological
materials consumed, and all of the biological wastes generated, by that person in a given
year. These materials and wastes each demand ecologically productive areas, such as
cropland to grow potatoes, or forest to sequester fossil carbon dioxide emissions. All of
these materials and wastes are then individually translated into an equivalent number of
global hectares.
Q.37) Consider the statements regarding Coral bleaching
1. It is the loss of zooxanthellae through either expulsion or loss of algal pigmentation.
2. It happens due to increase in ocean temperature only
3. All the corals undergoing bleaching are dead corals
Select the correct code
a) 1 and 3
b) Only 2
c) Only 1
d) 2 and 3
Q.37) Solution (c)
First statement is true. Coral bleaching not only happens because of warm temperature but
can also happen due to Cold temperature. Not all bleaching events are due to warm water.
In January 2010, cold water temperatures in the Florida Keys caused a coral bleaching event
that resulted in some coral death. Water temperatures dropped 12.06 degrees Fahrenheit
lower than the typical temperatures observed at this time of year. Researchers will evaluate
if this cold-stress event will make corals more susceptible to disease in the same way that
warmer waters impact corals.
When a coral bleaches, it is not dead. Corals can survive a bleaching event, but they are
under more stress and are subject to mortality.

www.iasbaba.com

19

Environment-Ecology-Biodiversity- Subject wise compilation of 60 Day Plan- 2016


http://www.theaustralian.com.au/news/nation/great-barrier-reef-scientists-exaggeratedcoral-bleaching/news-story/99810c83f5a420727b12ab255256774b

Q.38) Which of the following are the features of newly notified E-Waste Management
Rules, 2016 by the Ministry of Environment, Forest and Climate Change?
1. These rules for the first time will bring the producers under Extended Producer
Responsibility (EPR).
2. For the first time, Compact Fluorescent Lamp (CFL) and other mercury containing lamp
brought under the purview of rules.
3. Provision for Pan India EPR Authorization by CPCB has been introduced replacing the
state wise EPR authorization.
4. Micro and Small industry sector as defined in Micro Small and Medium Developmental
Act, 2006 are completely exempted with EPR Responsibility.
Choose the appropriate code:
a) 1 and 4 only
b) 1, 3 and 4 only
c) 1, 2 and 3 only
d) All of the above
Q.38) Solution (d)
Explanation:
All the given statements are correct and self-explanatory
India has emerged as the worlds fifth largest electronic waste (e-waste) producer.
Exemption continues for micro enterprises; however small enterprises, which have been
referred as one of the major source of generation of e-waste, have been included in the
rules for responsibility as manufacturer, without burdening them with EPR responsibility
as applicable to Producers.
Source: http://pib.nic.in/newsite/PrintRelease.aspx?relid=138319
Q.39) Select the incorrect pair from the below given important environment related days
and its 2016 theme:
1. World Oceans Day (8th June every year) : : Healthy oceans, healthy planet
2. World Environment Day (5th June every year) : : Go Wild for Life -- Zero tolerance for
the illegal trade in wildlife
3. Earth Day (22nd April every year) : : Wetlands for our Future Sustainable Livelihoods
4. None of the above
Q.39) Solution (c)

www.iasbaba.com

20

Environment-Ecology-Biodiversity- Subject wise compilation of 60 Day Plan- 2016


Explanation:
Wetlands for our Future Sustainable Livelihoods is the 2016 Theme of World
Wetland Day (which is celebrated on 2nd February every year)
Every year Earth Day is being observed on the 22nd April to build support for
environmental protection around the world. This year is 46th anniversary of Earth Day.
Earth Days 2016 Theme: Trees for the Earth!.
The theme signifies that trees are crucial for combating climate change as they absorb
harmful levels of Carbon dioxide (CO2) from the atmosphere. This years Earth Day
coincides with the signing of the historic Paris Agreement on Climate Change.
The World Oceans Day is being observed globally on 8th June every year to raise global
awareness of the current challenges faced by the international community in connection
with the oceans.
2016 Theme: Healthy oceans, healthy planet. The theme highlights main urgent need
to curb the plastic pollution.
Do you know?
Every year May 22 is being observed as International Biodiversity Day (IDB) across the
world to promote conservation and sustainable use of biodiversity.
2016 Theme: Mainstreaming Biodiversity; Sustaining People and their Livelihoods. The
theme highlights the role of biodiversity in underpinning development. The United
Nations (UN) also has highlighted the importance of biodiversity as an important crosscutting issue in in the Sustainable Development Goals (SDGs).
The day commemorates the adoption of Agreed Text of the Convention of Biological
Diversity (CBD) on 22 May 1992 by the Nairobi Final Act.
Source: Various newspaper articles and environment related websites
Q.40) Recently, the Union Government has launched the Centre for Environmental Health
in New Delhi. Consider the following statements in regard to it:
1. The centre will assess the impact of environment-related problems like climate change,
air pollution, pesticide use and sanitation on health.
2. It is a joint initiative of the Bombay Natural History Society (BNHS) and Public Health
Foundation of India (PHFI).
3. The initiative was launched by Union Minister for Health and Family Welfare.
Which of the statements given above is/are correct?
a) 2 and 3 only
b) 1 and 2 only
c) 1 and 3 only
d) All of the above

www.iasbaba.com

21

Environment-Ecology-Biodiversity- Subject wise compilation of 60 Day Plan- 2016


Q.40) Solution (c)
Explanation:
It is a joint initiative of the Tata Institute of Social Science (TISS) and Public Health
Foundation of India (PHFI).
Centre for Environmental Health will assess the impact of environment-related problems
like climate change, air pollution, pesticide use and sanitation on health.
The centre will conduct research across wide range of environmental health issues,
including water, chemical exposure and hygiene.
It will also establish a policy engagement platform with regular meetings with the civil
society, government, academia and private sector to develop strategies for better
implementation of the eco-friendly policies.
Source:
http://www.uniindia.com/nadda-launches-centre-for-environmentalhealth/india/news/488858.html
Q.41) Which among the following statements are correct in regard to recently adopted
historic
Paris Climate Agreement at the 21st Conference of the Parties of the UNFCCC?
1. The agreement facilitates enforcement of global GHGs reduction measures, adaptation
and finance in the post-2020 i.e. in post Kyoto Protocol scenario.
2. All member countries agreed to work to limit global temperature rise to well below 2
degrees Celsius, and to strive for 1.5 degrees Celsius.
3. All member countries agreed to take into account the principle of equity (climate justice)
and common but differentiated responsibilities (CBDR) and respective capabilities.
4. Developed countries will provide 100 billion dollars annually to the developing
counterparts beginning in 2020 and it would increase with time.
Choose the appropriate code:
a) 1 and 4 only
b) 1, 3 and 4 only
c) 1 and 3 only
d) All of the above
Q.41) Solution (d)
Explanation:
Self explanatory All the given statements are correct
Q.42) If a private company wants to set up an industrial or infrastructure projects in a
forest land, then:
1. The Forest (Conservation) Act, 1980 (FCA) governs diversion or use of forest land for
non-forest purposes such as industrial or infrastructure projects.

www.iasbaba.com

22

Environment-Ecology-Biodiversity- Subject wise compilation of 60 Day Plan- 2016


2. A company diverting forest land must provide alternative land for taking up
compensatory afforestation.
3. For the afforestation purpose, the company should pay for planting new trees in the
alternative land provided to the state.
4. The loss of forest ecosystem must also be compensated by paying for net present
value of forest (NPV).
Which of the statements given above is/are incorrect?
a) None of the above
b) 1 and 4 only
c) 4 only
d) 1 only
Q.42) Solution (a)
Explanation:
Self explanatory All the given statements are correct
Q.43) Ministry of Environment, Forest and Climate Change recently released a new Fourcolor Classification Scheme for industries based on their pollution potential. Select the
correctly matched new categorization system:
1. Red category: Pollution Index (PI) score of 60 and above.
2. Orange category: PI score of 41 to 59.
3. Yellow category: PI score of 21 to 40.
4. Green category: PI score below and upto 20.
Choose the appropriate code
a) 1 and 2 only
b) 1, 3 and 4 only
c) 1 and 4 only
d) 1, 2 and 4 only
Q.43) Solution (a)
Explanation:
Green category: PI score of 21 to 40.
White category: PI score below and upto 20.
There is no Yellow category.
Q.44) Recently, the Union cabinet gave its formal approval for adopting the Statute of the
South Asia Wildlife Enforcement Network (SAWEN). Select the incorrect statement from
the following in regard to SAWEN:

www.iasbaba.com

23

Environment-Ecology-Biodiversity- Subject wise compilation of 60 Day Plan- 2016


a) SAWEN is regional inter-governmental wildlife law enforcement support body launched
in January, 2011 in Paro, Bhutan.
b) SAWEN aims to strengthen, promote and co-ordinate regional co-operation for curbing
illegal wildlife trade that threatens the wild flora and fauna of South Asia.
c) Except Pakistan, all other South Asian countries will be the part of SAWEN.
d) None of the above
Q.44) Solution (c)
Explanation:
All 8 South Asian countries will be the part of SAWEN (i.e. including Pakistan)
Q.45) Consider the following statements regarding Agenda 21
1. It is a non-binding and voluntary implementation plan of United Nations with regard to
sustainable development
2. United Nation Convention to Combat Desertification (UNCCD), a legally binding
international agreement stemmed from Agenda 21
3. Agenda 21 also mentions about protection of atmosphere
Select the appropriate code
a) 1 and 2 Only
b) 2 and 3 Only
c) 1 and 3 Only
d) 1, 2 and 3
Q.45) Solution (d)
All the statements are correct.
Q.46) Consider the following International Conventions
1. Aarhus Convention
2. Geneva Convention
3. Stockholm Convention
4. Rotterdam Convention
5. Basel Convention
6. Bonn Convention
Which of the above are related to Hazardous Substances?
a) 1, 2, 3, 5 and 6
b) 2, 3, 4 and 5
c) 1, 2, 3, 4, 5 and 6
d) 1, 2, 3, 4 and 5

www.iasbaba.com

24

Environment-Ecology-Biodiversity- Subject wise compilation of 60 Day Plan- 2016


Q.46) Solution (b)
Aarhus Convention- The UNECE Convention on Access to Information, Public Participation in
Decision-making and Access to Justice in Environmental Matters, usually known as the
Aarhus Convention
Bonn Convention- The Convention on the Conservation of Migratory Species of Wild
Animals -- more commonly abbreviated to just the Convention on Migratory Species (CMS)
or the Bonn Convention -- aims to conserve terrestrial, marine and avian migratory species
throughout their range.

Q.47) Amur falcons has one of the longest and most fascinating migratory paths in the
avian kingdom. Consider the following statements regarding Amur falcons:
1. It is one of the largest raptor species.
2. It migrates from Mongolia to South Africa via India.
3. It is amongst the critically endangered species and very few left in wild.
4. It considered a delicacy in Nagaland and widely hunted during their short stay in India.
Select the correct statements from above:
a) 1 and 3
b) 2 and 4
c) 3 only
d) All of the above
Q.47) Solution (b)
It is a small raptor, of the size of a pigeon. It covers one of the largest migratory distances
from Mongolia and Russia to South Africa. It is not a critically endangered species and is
found in large numbers. However, due to indiscriminate killing, their numbers are rapidly
coming down.
It is considered as a delicacy in Nagaland and widely hunted and sold in the local market.
Q.48) Endosulfan has been used as a pesticide in agriculture for a long time. India has
agreed to phase out completely the use and manufacture of endosulfan by 2017. Which of
the following statements are correct about endosulfan?
1. It is not pest specific and it can negatively impact the populations of beneficial insects.
2. Endosulfan is acutely neurotoxic to both insects and mammals.
3. Endosulfan has high potential to bio-accumulate and bio-magnify.
4. Endosulfan is banned across the world under Stockholm convention.
5. Endosulfan is the cause of death of Indian vultures.
Select the code from the following:
a) 1, 2 and 3

www.iasbaba.com

25

Environment-Ecology-Biodiversity- Subject wise compilation of 60 Day Plan- 2016


b) 1, 3 and 4
c) 2, 3, 4 and 5
d) All of the above
Q.48) Solution (a)
Endosulfan ban is being discussed under Stockholm convention for Persistent Organic
Pollutants but nothing is yet decided.
Cause of death of Indian vultures is the use of diclofinac.
Q.49) Government of India has being trying to protect the critically endangered Great
Indian Bustard. Which of the following statement are correct about Great Indian Bustard?
1. To protect the bird and create awareness, this has been made the state bird of Haryana.
2. It is a flight less bird.
3. It is found in arid and semi-arid grassland.
Select the code from the following:
a) All of the above
b) 1 and 2
c) 2 and 3
d) 3 only
Q.49) Solution (d)
Great Indian Bustard is one of the heaviest flying birds. It has been declared as the state bird
of Rajasthan.
Q.50) Which of the following gas emissions are checked when pollution check of vehicles
are done in India?
1. Carbon mono Oxide
2. Hydrocarbons
3. Carbon di-oxide
4. Sulphur Oxides
Select the correct code from the following:
a) 1 and 2
b) 3 and 4
c) 1,2 and 4
d) All of the above
Q.50) Solution (a)
Only the amount of Carbon mono Oxide and Hydrocarbons is checked in exhaust from the
vehicle in pollution check.

www.iasbaba.com

26

Environment-Ecology-Biodiversity- Subject wise compilation of 60 Day Plan- 2016

Q.51) India is amongst the first and foremost countries to have a forest policy. Although
the first policy of 1894 was designed on how to exploit the forest resources, constant
revisions were made in the policy after independence of India. The current policy was
made in 1988, known as The National Forest Policy 1988. Which of the following
statements are incorrect about this policy?
1. Maintenance of environmental stability through the preservation and restoration of
ecological balance.
2. Conservation of forests as a national heritage with vast varieties of flora and fauna.
3. Control of soil erosion and denudation in catchment areas of rivers, lakes and reservoirs.
4. Check on the extension of sand-dunes in desert areas.
5. To meet the needs of fuel-wood, fodder and minor forest products for the rural and
tribal people.
6. Encouragement of efficient utilization of forest produce and optimum substitution of
wood.
Select the code from the following:
a) 1,2 and 3
b) 4,5 and 6
c) 5 and 6
d) None of the above
Q.51) Solution (d)
Since all the statements are part of the policy, none of the above statements are incorrect.
The National Forest Policy 1988:
The main emphasis of the National Forest Policy 1988 is on the protection, conservation,
regeneration and development of forests. The main points of the National Forest Policy
1988 are:
(a) Maintenance of environmental stability through the preservation and restoration of
ecological balance.
(b) Conservation of forests as a national heritage with vast varieties of flora and fauna.
(c) Control of soil erosion and denudation in catchment areas of rivers, lakes and
reservoirs.
(d) Check on the extension of sand-dunes in desert areas of Rajasthan and along seacoasts.
(e) Substantial increase in forest cover through massive afforestation and social forestry
programmes.
(f) To meet the needs of fuel-wood, fodder and minor forest products for the rural and
tribal people.
(g) Augment the productivity of the forests to meet national needs.

www.iasbaba.com

27

Environment-Ecology-Biodiversity- Subject wise compilation of 60 Day Plan- 2016


(h) Encouragement of efficient utilisation of forest produce and optimum substitution of
wood.
(i) Steps to create massive movement of people with the involvement of women folk to
achieve these objectives and to minimise pressure on existing forests.
(j) Involvement of people in forest management under joint forest management.
Q.52) Since the solid waste generation is rising in urban areas of India, many waste to
energy plants are being started. They serve the dual purpose of waste disposal as well as
generation of electricity. Which of the following statements are correct about waste to
energy generation?
1. The cost of waste to energy is somewhat higher than other renewable sources.
2. The chimneys of incinerators include acid gases, nitrogen oxide, heavy metals,
particulates, and dioxin, which are carcinogenic.
3. The efficiency of waste to energy plants is very low.
Select the correct code from the following:
a) 1 and 2
b) 2 and 3
c) 1 and 3
d) All of the above
Q.52) Solution (d)
While the Indian Governments own figures would suggest that the cost of waste to energy
is somewhat higher than other renewable sources, it is still an attractive option, as it serves
a dual role of waste disposal and energy production.
Smoke and ash emitted by the chimneys of incinerators include acid gases, nitrogen oxide,
heavy metals, particulates, and dioxin, which are carcinogenic. While incineration pollution
control technology is evolving to reduce these pollutants, it has been found that even with
controls in place, some remaining dioxin still enters the atmosphere.
Q.53) Consider the following statements regarding Minamata Convention:
1. It is an international treaty designed to protect human health and the environment from
anthropogenic emissions and releases of mercury.
2. It is named after Japanese city Minamata which went through devastating incident of
mercury poisoning.
3. India has ratified the Minamata convention.
4. The developed countries have promised to help financially, technically, and
technologically to developing countries.
Which of the above statements are correct?
a) 1, 2 and 3

www.iasbaba.com

28

Environment-Ecology-Biodiversity- Subject wise compilation of 60 Day Plan- 2016


b) 2, 3 and 4
c) 1, 2 and 4
d) All of the above
Q.53) Solution (c)
India is a signatory to Minamata convention but it has still not ratified it.
Q.54) Consider the following:
International Conventions
1. Convention of Nuclear Safety
2. United Nations Convention on the Law of the Sea
3. Convention on Persistent Organic Pollutants
4. Convention on the Control of Transboundary

Location
Vienna
Montego Bay
Stockholm
Basel

Movements of Hazardous Wastes and their Disposal


Which of the above are correctly matched?
a) 1,2 and 3
b) 2,3 and 4
c) 1,3 and 4
d) All of the above
Q.17) Solution (d)
Self explanatory.
Q.54) Consider the following statements:
1. Mixing fly ash in soil can help farmers increase production of crops and vegetables
2. Fly ash can be mixed with cement thus reducing the cost of construction.
3. Fly ash can cover the surface of plant leaves and thus helps in increasing nutrient
capacity.
4. Fly ash bricks are light weight and offer high strength and durability.
5. Use of fly-ash instead of lime in agriculture can reduce net CO2 emission and also reduce
global warming.
Which of the above statements are correct w.r.t Fly Ash?
a) 1, 2 , 3 and 4
b) 2, 3, 4 and 5
c) 1, 2, 4 and 5
d) All of the above
Q.54) Solution (c)

www.iasbaba.com

29

Environment-Ecology-Biodiversity- Subject wise compilation of 60 Day Plan- 2016


Fly ash in atmosphere acts as a pollutant. It can cover the leaf surface by making a thin layer
which reduces photosynthesis and productivity of plants. But its use in soils shows different
result. Fly ash is a resourceful material and can be effectively utilized as soil modifier in large
quantity and micro fertilizer in converting wasteland (barren land, rocky nature, sandy and
water logged soil, highly alkali and acidic soil etc.) into agriculturally productive land. "Best
thing about fly ash is, that it retains water in ground and helps bacterial actions to take
place to cultivate good quality of crops or vegetables.
Refer to these
http://scialert.net/fulltext/?doi=ajar.2010.1.14
http://timesofindia.indiatimes.com/city/bhopal/Fertilizer-costly-fly-ash-can-workmagic/articleshow/33835877.cms
Q.55) Which of the following is not concerned with the Wildlife Protection Act?
1. National Parks
2. Wildlife Sanctuaries
3. Conservation Reserves
4. Biosphere Reserves
5. Community Reserves
6. Tiger Reserves
Select the correct code
a) 3,4 and 5
b) Only 4
c) 4 and 5
d) None of the above
Q.55) Solution (b)
National Parks, Wildlife Sanctuaries, Conservation Reserves, Community Reserves and Tiger
Reserves are established as per provisions of Wildlife Protection Act, there is no law as such
under which Biosphere Reserves are established.
Q.56) Consider the following statements with respect to India Biodiversity Award
1. It is a joint initiative by Ministry of Environment, Forest and Climate Change, National
Biodiversity Authority and United Nations Environment Programme
2. The categories for the award are aligned with Biological Diversity Act, 2002
3. Satpura Tiger Reserve won the India Biodiversity Award this year
Which of the following statements is/are incorrect?
a) Only 1
b) Only 3
c) 1 and 3

www.iasbaba.com

30

Environment-Ecology-Biodiversity- Subject wise compilation of 60 Day Plan- 2016


d) All of the above
Q.56) Solution (c)
It is joint initiative of the Union Ministry of Environment, Forest and Climate Change
(MoEFCC), National Biodiversity Authority (NBA) and United Nations Development
programme (UNDP). http://envfor.nic.in/sites/default/files/IBA%20Brochure%2024-Aug2015.pdf
Pakke Tiger Reserve in East Kameng district of Arunachal Pradesh has received the 'India
Biodiversity
Award
2016'

http://timesofindia.indiatimes.com/home/environment/Arunachal-tiger-reserve-bagsbiodiversity-award/articleshow/52467241.cms
Q.57) Recently Himachal Pradesh fisheries department has started an artificial
propagation programme for rehabilitation and conservation of Golden Mahseer fish.
Consider the following statements with respect to Golden Mahseer Fish.
1. It is the longest-living freshwater fish
2. It is native to mountain and sub-mountain regions
3. It inhabit only rivers
4. It is an omnivore
Select the correct code
a) 1,2 and 3
b) 1,2 and 4
c) 1 and 2
d) All of the above
Q.57) Solution (b)
Golden Mahseer Fish is the longest-living freshwater fish and is native to mountain and
sub-mountain regions.
It inhabit both rivers and lakes and is an omnivore
http://timesofindia.indiatimes.com/home/environment/flora-fauna/Himachal-Pradeshpropagating-mahseer-fish-for-conservation/articleshow/52348942.cms
https://en.wikipedia.org/wiki/Mahseer
Q.58) One of the most accepted definitions of sustainable development is Development
that meets the needs of the present without compromising the ability of future
generations to meet their own needs." This definition is given by which of the following
commissions?
a) World Commission on Environment and Development
b) Environment and energy commission

www.iasbaba.com

31

Environment-Ecology-Biodiversity- Subject wise compilation of 60 Day Plan- 2016


c) European Commission on environment
d) Sustainable and Legacy Commission
Q.58) Solution (a)
The definition was given on 1987 by World Commission on Environment and Development,
popularly known as the Brundtland Commission.
Q.59) Consider the following statements:
1. Agenda 21 is a binding action plan of the United Nations with regard to Sustainable
Development.
2. It is a product of Earth Summit held at Rio de Jeneiro in 1992.
3. 21 in Agenda 21 refers to 21 points of the document that has been pledged for
sustainable development.
Which of the above statements are incorrect?
a) 1 and 2
b) 2 and 3
c) 1 and 3
d) None of the above
Q.59) Solution (c)
Agenda 21 is a non-binding, voluntarily implemented action plan of the United Nations with
regard to sustainable development. It is a product of the Earth Summit (UN Conference on
Environment and Development) held in Rio de Janeiro, Brazil, in 1992. It is an action agenda
for the UN, other multilateral organizations, and individual governments around the world
that can be executed at local, national, and global levels. The "21" in Agenda 21 refers to the
21st Century. It has been affirmed and had a few modifications at subsequent UN
conferences.
Q.60) Heat from the earth can be used as an energy source in many ways, from large and
complex power stations to small and relatively simple pumping systems. This heat energy
is known as geothermal energy. Consider the following statements regarding geo thermal
energy:
1. The predominant source of the Earths heat is the gradual decay of radioactive isotopes.
2. It has extensive global distribution and it is accessible in both developed as well as under
developed nations.
3. It has low emission of sulphur, CO2 and other greenhouse gases.
4. It is independent of external supply and demand effects and fluctuations in exchange
rates.
Which of the above statements are correct?

www.iasbaba.com

32

Environment-Ecology-Biodiversity- Subject wise compilation of 60 Day Plan- 2016


a)
b)
c)
d)

1,2 and 3
2,3 and 4
1,2 and 4
All of the above

Q.60) Solution (d)


Heat from the earth can be used as an energy source in many ways, from large and complex
power stations to small and relatively simple pumping systems. This heat energy, known as
geothermal energy, can be found almost anywhereas far away as remote deep wells in
Indonesia and as close as the dirt in our backyards.
Many regions of the world are already tapping geothermal energy as an affordable and
sustainable solution to reducing dependence on fossil fuels, and the global warming and
public health risks that result from their use.
The general characteristics of geothermal energy that make it of significant importance for
both electricity production and direct use include:
Extensive global distribution; it is accessible to both developed and developing
countries.
Environmentally friendly nature; it has low emission of sulphur, CO2 and other
greenhouse gases.
Indigenous nature; it is independent of external supply and demand effects and
fluctuations in exchange rates.
Independence of weather and season.
Contribution to the development of diversified power sources.
Q.61) Solar energy is radiant light and heat from the Sun that is harnessed using a range of
ever-evolving technologies. Solar power is the conversion of sunlight into electricity,
either directly using photovoltaics (PV), or indirectly using concentrated solar power
(CSP). Consider the following statements regarding Solar Power Technology:
1. Photovoltaic cells use the ultraviolet radiations to convert solar energy into electric
current.
2. Semiconductors (usually silicon) are used to manufacture solar panels.
3. Concentrated Solar power use infrared radiations to heat the water and rotate turbines
with the generated steam.
4. Concentrated Solar Power systems generally use a huge convex lens to concentrate
energy at its focus.
Which of the above statements are correct?
a) 1 and 2
b) 3 and 4
c) 2 and 3
d) 1,2 and 3

www.iasbaba.com

33

Environment-Ecology-Biodiversity- Subject wise compilation of 60 Day Plan- 2016

Q.61) Solution (c)


Photovoltaic cells use photons or light radiations to produce electric current.
Concentrated Solar Power systems usually use large set of plane mirrors or concave mirrors
to focus sunlight.
Q.62) Wind energy has a very high potential in India. Indian government has already
proposed a National Wind Mission on the lines of National Solar Mission. Which of the
following statements are correct Regarding Wind energy in India?
1. In 2015-16, Indian Government fell short of achieving its target of wind energy
production.
2. The installed capacity of wind power in India is more than 25000MW.
3. East and North East region of India has no grid connected wind power plant.
4. The highest wind power production in India is in Gujarat.
Select the code from the following:
a) None of the above
b) 2 and 3
c) 1 and 4
d) 1,2 and 3
Q.62) Solution (b)
As of 31 March 2016 the installed capacity of wind power in India was 26,769 MW, mainly
spread across South, West and North regions. East and North east regions have no grid
connected wind power plant as of March, 2015 end. No offshore wind farm utilizing
traditional fixed-bottom wind turbine technologies in shallow sea areas or floating wind
turbine technologies in deep sea areas are under implementation. However, an Offshore
Wind Policy was announced in 2015 and presently weather stations and LIDARs are being
set up by NIWE at some locations.
Tamil Nadu has the highest installed capacity of wind power in India.
Q.63) Consider the following statements regarding National Mission on Sustainable
Habitats:
1. It is one of the missions under National Action Plan on Climate Change.
2. It works for the development and protection of natural habitats, especially of those
species which are at the verge of extinction due to habitat loss.
3. The mission works on development of green corridors to connect protected areas of
India.
Which of the above statements are incorrect?
a) 1 and 2

www.iasbaba.com

34

Environment-Ecology-Biodiversity- Subject wise compilation of 60 Day Plan- 2016


b) 2 and 3
c) 3 only
d) None of the above
Q.63) Solution (b)
The national mission on sustainable habitat approved by the Prime Minister. It is one of the
nine missions under National Action Plan on Climate Change and aims to make cities
sustainable through improvements in energy efficiency in buildings, management of solid
waste & shift to public transport.
Q.64) Carbon neutrality refers to achieving net zero carbon emissions by balancing a
measured amount of carbon released with an equivalent amount sequestered or offset, or
buying enough carbon credits to make up the difference. Which of the following countries
are carbon neutral?
a) Denmark
b) Bhutan
c) Iceland
d) Sweden
Q.64) Solution (b)
Two countries have achieved carbon neutrality: Vatican city and Bhutan
Several countries have pledged carbon neutrality, including: British Columbia (Canadian
province), Costa Rica, Iceland, Maldives, New Zealand, Norway, Tuvalu and Sweden.
Q.65) Buildings contribute maximum to the green house gas emissions in the world. To
reduce the carbon foot print of buildings, the concept of green buildings was given.
Consider the following regarding Green Buildings:
1. Green building refers to both a structure and the using of processes that are
environmentally responsible and resource-efficient throughout a building's life-cycle.
2. GRIHA, an acronym for Green Rating for Integrated Habitat Assessment, is the National
Green Building Rating System of India.
3. GRIHA has been developed by TERI.
Which of the above statements are correct?
a) 1 and 2
b) 2 and 3
c) 1 and 3
d) All of the above
Q.65) Solution (d)

www.iasbaba.com

35

Environment-Ecology-Biodiversity- Subject wise compilation of 60 Day Plan- 2016


Green building (also known as green construction or sustainable building) refers to both a
structure and the using of processes that are environmentally responsible and resourceefficient throughout a building's life-cycle: from siting to design, construction, operation,
maintenance, renovation, and demolition. In other words, green building design involves
finding the balance between homebuilding and the sustainable environment. This requires
close cooperation of the design team, the architects, the engineers, and the client at all
project stages. The Green Building practice expands and complements the classical building
design concerns of economy, utility, durability, and comfort.
Leadership in Energy and Environmental Design (LEED) is a set of rating systems for the
design, construction, operation, and maintenance of green buildings which was Developed
by the U.S. Green Building Council.
GRIHA, an acronym for Green Rating for Integrated Habitat Assessment, is the National
Rating System of India. TERI took the responsibility of popularizing green building by
developing a tool for measuring and rating a building's environmental performance in the
context of India's varied climate and building practices.
Q.66) Which of the following are the result of modern agriculture?
1. Nitrate pollution
2. Eutrophication
3. Biomagnification
4. Ozone Depletion
5. Salinisation
Select the code from the following:
a) 1,2 and 3
b) 1,2,3 and 5
c) 2,3,4 and 5
d) All of the above
Q.66) Solution (b)
Direct question.
Ozone depletion is not the result of agricultural activities
Many of you have mentioned with the sources that there are some modern pesticides
containing compounds like Methyl Bromide etc. So the answer of this question would be (d)
However, the sources which I referred to make this question, did not mention about Ozone
depleting substance being used. So I think there is an ambiguity. Many of the reports in
newspapers are recent researches which are most of the time non- conclusive. So we
cannot take them as a parameter and believe what they are saying. Lets hope we dont get
this ambiguity in the exam.

www.iasbaba.com

36

Environment-Ecology-Biodiversity- Subject wise compilation of 60 Day Plan- 2016

Q.67) Which of the following statements correctly explains an Urban Heat Island?
a) A city or a metropolitan area, which is significantly warmer than the surrounding rural
area, due to anthropogenic activities.
b) An island with sudden increase in urbanization activity, increasing its average
temperature.
c) A volcanic island where people have settled due to population pressure on the
mainland.
d) None of the above
Q.67) Solution (a)
An urban heat island (UHI) is a city or metropolitan area that is significantly warmer than its
surrounding rural areas due to human activities. The temperature difference usually is larger
at night than during the day, and is most apparent when winds are weak. The main cause of
the urban heat island effect is from the modification of land surfaces. Waste heat generated
by energy usage is a secondary contributor. As a population center grows, it tends to expand
its area and increase its average temperature. The less-used term heat island refers to any
area, populated or not, which is consistently hotter than the surrounding area.
Q.68) Consider the following
1. Mosses
2. Litchens
3. Microbes
Which among the above is not a pioneer species?
a) 1 and 2 only
b) 2 and 3 only
c) 3 only
d) None
Q.68) Solution (d)
Lichens, mosses, microbes are some of the pioneer species in primary succession. In primary
succession on a terrestrial site the new site is first colonised by a few hardy pioneer species
that are often mosses, lichens, microbes.
These new conditions may be conductive to the establishment of additional organisms that
may subsequently arrive at the site.
Q.69) A Biotic interaction in which one species is harmed and other is unaffected is
a) Competition
b) Mutualism
c) Commensalism

www.iasbaba.com

37

Environment-Ecology-Biodiversity- Subject wise compilation of 60 Day Plan- 2016


d) Amensalism
Q.69) Solution (d)
Amensalism is a biotic interaction in which one species is harmed and other is unaffected.
For ex- a large tree shades a small plant, retarding the growth of small pants. The small
plants have no effect on the large tree.
Q.70) Refer the diagrams below. Which diagram best represents the average monthly
temperature changes during a year for a tundra in the Northern Hemisphere?

Q.70) Solution (a)


Explanation:
Diagram (a) best represents the average monthly temperature changes during a year for
a tundra in the Northern Hemisphere.
A tundra is a level or undulating, treeless plain characteristic of arctic and subarctic
regions with long, cold winters and short, cool summers.
Q.71) Where do the fungi obtain nutrients and energy?

www.iasbaba.com

38

Environment-Ecology-Biodiversity- Subject wise compilation of 60 Day Plan- 2016


a)
b)
c)
d)

The Sun
The primary producers only
The consumers only
All members of the community

Q.71) Solution (d)


Explanation:
Fungi, as a group, are heterotrophic organisms with an absorptive mode of nutrition. They
can obtain energy and nutrients from all members of the community.
Q.72) A lichen is best characterized as a symbiosis between which of the following?
a) A liverwort and a fungus
b) A moss and a liverwort
c) An alga and a moss
d) An alga and a fungus
Q.72) Solution (d)
Explanation:
A lichen represents a symbiotic relationship between an alga and a fungus.
Q.73) Of the following, the major cause of infant mortality worldwide is
a) starvation
b) waterborne diseases
c) toxic chemicals
d) nuclear radiation
Q.73) Solution (b)
Explanation:
Water is used for sewage systems, drinking, washing activities, and food production.
Cross contamination of drinking and washing water with sewage is a serious problem
worldwide that leads to the spread of waterborne diseases and infant mortality.
Q.74) All of the following are likely to increase after large areas of tropical rain forests are
cut down EXCEPT the
a) species diversity of the areas
b) erosion by rivers fiowing through the areas
c) rate of nutrient loss from the areas
d) average surface temperature of the soil in the areas
Q.74) Solution (a)
Explantion:

www.iasbaba.com

39

Environment-Ecology-Biodiversity- Subject wise compilation of 60 Day Plan- 2016


The correct answerthe exceptionis (a). The continuing loss of the diverse habitats found
in tropical rain forests will cause a decrease in species diversity, not an increase.
Q.75) Observations of lakes in areas with granitic bedrock indicate that the lakes are
becoming depleted of living organisms. The primary cause is considered to be
a) nuclear waste
b) lowered water levels
c) acid rain
d) garbage dumping
Q.75) Solution (c)
Explanation:
Rain combines with various types of air pollution to form acid rain.
Lakes formed in limestone bedrock can neutralize the acidity of the rain, but lakes with
granitic bedrock have limited capacity to do so.
The increasing acidity of such lakes has had a negative impact on the organisms normally
inhabiting them.
Q.76) Which of the following is a true statement about the flow of energy in an
ecosystem?
a) Smaller organisms need less energy per gram of body weight than do larger organisms.
b) Energy transfer between organisms normally involves conservation of heat energy.
c) Energy flow between trophic levels is inefficient.
d) Chemical energy is converted into radiant energy, which is then converted into chemical
energy at the next trophic level.
Q.76) Solution (c)
Explanation:
Biologists estimate that approximately 90 percent of the energy is lost between levels of
an energy pyramid. Thus, it is true that energy is transferred between trophic levels
inefficiently.
Q.77) Which of the following is most likely the result of decreasing levels of ozone in the
stratosphere?
a) A decrease in the levels of smog in major cities
b) A decrease in the rate of global warming
c) An increase in the occurrence of skin cancer in humans
d) An increase in photosynthetic activity of phytoplankton
Q.78) Solution (c)
Explanation:

www.iasbaba.com

40

Environment-Ecology-Biodiversity- Subject wise compilation of 60 Day Plan- 2016

Ground-level ozone is involved with smog formation, not ozone in the stratosphere.
While tropospheric ozone is considered a greenhouse gas, stratospheric ozone is not
thought to have a significant effect on global warming.
Decreasing levels of stratospheric ozone result in increased levels of ultraviolet radiation
reaching the ocean, which may inhibit phytoplankton photosynthetic activity in surface
waters.
The increased levels of ultraviolet radiation can also negatively affect human health.

Q.79) Weather events characterized by heavy rain, thunderstorms, and a sharp


temperature drop followed by clearing are most likely related to which of the following?
a) The passage of a cold front
b) The passage of a warm front
c) The action of a stationary front
d) The heat island effect
Q.79) Solution (a)
Explanation:
The passage of a cold front is often accompanied by rain and sometimes thunder.
There is generally a sharp decrease in temperature while the front is passing and a
continuing decrease in temperature after it has passed.
Passage of the front also typically brings a decrease in precipitation followed by clearing.
The other choices are not consistent with this set of conditions.
Q.80) Which of the following substances is found in the runoff of chemically fertilized
lawns and is most likely to cause eutrophication in streams and lakes?
a) Sodium
b) Carbonate
c) Phosphate
d) Sulfate
Q.80) Solution (c)
Explanation:
Excess phosphates from lawn fertilizers can be carried by runoff into streams and lakes,
causing rapid plant growth and algal blooms. Sodium, carbonate, and sulfate are less
likely to be limiting nutrients and supplied by chemical lawn fertilizers.
Q.81) Methane, a gas that can be trapped and used for energy, is most likely to be
produced by which of the following?
a) Decomposition of organic materials in landfills
b) Burning of organic materials in incinerators
c) Heating of inorganic materials in a high oxygen atmosphere
www.iasbaba.com

41

Environment-Ecology-Biodiversity- Subject wise compilation of 60 Day Plan- 2016


d) Recycling of metals in recycling plants
Q.81) Solution (a)
Explanation:
Methanogenic microorganisms can produce methane from organic material in landfills.
Burning of organic material will not produce methane. Heating of inorganic materials in
a high-oxygen atmosphere or the recycling of metals are not likely to produce significant
methane.
Q.82) In the discussion of the nature of and solution to environmental problems, it is most
important for students to understand that
a) science is constantly advancing, and technology will be developed to solve current
problems
b) changes in the abiotic portion of the biosphere are self-correcting and result in evolution
of the biotic portion of the biosphere
c) once pollution has been stopped, communities will revert to their preindustrial status
d) cultural, political, and economic issues must all be considered, in addition to scientific
issues
Q.82) Solution (d)
Explanation:
Understanding and finding solutions to environmental problems requires a
consideration of scientific, economic, political, and cultural issues.
Humans cannot assume that advances in science and technology alone will solve current
environmental problems.
There is no evidence that organisms will be able to evolve and self-correct in response
to the rapidly changing environment.
Since a number of organisms have gone extinct and large amounts of habitat have been
destroyed, simply stopping pollution cannot guarantee that communities will revert to
their preindustrial status.
Q.83) Which of the following hypotheses is LEAST amenable to verification by
experimentation?
a) Using compound X to control insect pests that damage crops also reduces the
population levels of pollinator insects.
b) Building a new factory in a town with high unemployment is more important than
preserving the habitat the factory will destroy.
c) Applying chemical fertilizers above recommended amounts increases the concentration
of inorganic nutrients in runoff.
d) Humans and chimpanzees share some of the same genes.

www.iasbaba.com

42

Environment-Ecology-Biodiversity- Subject wise compilation of 60 Day Plan- 2016


Q.83) Solution (b)
Explanation:
The correct answerthe LEAST amenable to verification by experimentationis choice
(B).
The hypothesis that jobs are more important than habitat cannot be tested by
experimentation.
The effects of chemicals on insects and of fertilizers on nutrients in runoff, as well as the
incidence of the same genes occurring in different species, can all be measured and
tested by experimentation.
Q.84) Consider the following
1. Sikkim
2. Assam
3. Arunachal Pradesh
4. Manipur
Arrange N-E States as per the percentage of Total Wetland Area in decreasing order
a) 2-3-4-1
b) 1-2-3-4
c) 1-3-2-4
d) 3-4-2-1
Q.84) Solution (a)
Read this report
http://envfor.nic.in/downloads/public-information/NWIA_National_brochure.pdf
Q.85) Global Environmental Facility (GEF) grants are available under five focal areas.
Identify the areas
1. Soil Degradation
2. International Waters
3. Climate Change
4. Energy
5. Land Degradation
6. Chemicals and Waste
7. Biodiversity
Select the correct code
a) 1, 3, 4, 5, 6
b) 2, 3, 4, 6 and 7
c) 1, 3, 5, 6 and 7
d) 2, 3, 5, 6 and 7

www.iasbaba.com

43

Environment-Ecology-Biodiversity- Subject wise compilation of 60 Day Plan- 2016

Q.85) Solution (d) The GEF grants are available under Biodiversity, Climate Change,
International Waters, Land degradation and chemicals and waste
Source- Ministry of Environment Annual Report- 2015-16
Q.86) Consider the following about Classical Smog
1. It occurs in hot humid climate
2. It is a mixture of smoke, fog, carbon dioxide and sulphur dioxide
3. It is also called as Oxidizing Smog
Select the correct codes
a) 1, 2 and 3
b) 2 and 3
c) 1 and 3
d) None
Q.86) Solution (d)
Occurrence- Cool Humid climate. Mixture of smoke, fog and SO2. It is reducing mixture so
called as Reducing Smog
NCERT- 11th Chemistry, chapter 14
Q.87) Consider the following statements
1. A Swamp is a Wetland that is forested
2. Myristica swamps are tropical fresh water swamp forests found in Kerala and Karnataka
3. Swamps serve vital roles in flood protection and nutrient removal
Select the correct code
a) 1 and 2
b) 2 and 3
c) 1 and 3
d) 1, 2 and 3
Q.87) Solution (d)
http://water.epa.gov/type/wetlands/swamp.cfm
Q.88) Thermal pollution is the dumping of heat into ecological systems like rivers, lakes,
ponds, ocean etc. which degrades the water quality. Consider the following statements
regarding the same.
1. It will decrease the metabolic activities of aquatic animals
2. Reproduction among aquatic animals will increase.
3. Bacterial activity will be limited in the water system

www.iasbaba.com

44

Environment-Ecology-Biodiversity- Subject wise compilation of 60 Day Plan- 2016


4. High temperature will increase the dispersion of oxygen into deeper waters thus leading
to aerobic condition.
Select the incorrect statement/s
a) 1, 2, 3, 4
b) 2 and 3
c) 3 and 4
d) None
Q.88) Solution (a)
The release of heated water into the aquatic bodies changes the average water temperature
and concentration of dissolved oxygen. Elevated temperature decreases the level of
dissolved oxygen in water which is harmful to aquatic animals like fishes, amphibians and
other aquatic organisms. High temperature limits oxygen dispersion into deeper waters,
leading to anaerobic conditions. It can lead to increased bacterial population. Several
aquatic species fail to reproduce at elevated temperature. The eggs of trout fail to hatch
while salmon does not spawn as higher temperature. Thermal pollution may also increase
the metabolic rate of aquatic animals, as enzyme activity, resulting in these organisms
consuming more food in a shorter time. An increased metabolic rate may result in fewer
resources causing a sharp decrease in a population.
A large increase in temperature can lead to the denaturation of enzymes. Decreased
enzyme activity in aquatic organisms can cause problems such as the inability to break down
fats, which leads to malnutrition.
Q.89) Consider the following regarding UN Man and Biosphere Programme (MAB)
1. This is a part of UNESCOs preservation of cultural heritage efforts
2. They form a part of Global Network under UN Biosphere World Network
3. Madrid Action Plan is associated with MAB

Select the correct code


a) 1 and 2
b) 2 and 3
c) 1 and 3
d) All
Q.89) Solution (b)
Launched in 1971, UNESCOs Man and the Biosphere Programme (MAB) is an
Intergovernmental Scientific Programme that aims to establish a scientific basis for the
improvement of relationships between people and their environments.

www.iasbaba.com

45

Environment-Ecology-Biodiversity- Subject wise compilation of 60 Day Plan- 2016


http://www.unesco.org/new/en/natural-sciences/environment/ecological-sciences/manand-biosphere-programme/
Q.90) Consider the following statements regarding Kyoto Protocol and Clean
Development Mechanism (CDM)
1. Kyoto Protocol is a legal but non-enforceable protocol
2. CDM is one of the flexible mechanisms defined in Kyoto Protocol
3. One crucial pre-condition of CDM is that the new projects should be in addition to those
which were already planned, so as to promote genuine new projects only which aim at
reducing emission. This is known as additionality clause.
Select the correct code
a) 1 and 2
b) 2 and 3
c) 1 and 3
d) All
Q.90) Solution (d)
Clean Development Mechanism (CDM) of UN It has been started in the aftermath of
Kyoto protocol under which Carbon Credits can be traded as per Marrakesh Accord. The
protocol came into effect in 2005 on the basis of Common, but Differentiated
Responsibilities. The principle of CBDR enshrined in UNFCCC article 4(7) was formally
operationalised through Kyoto Protocol. Under CDM, Certified Emission Reduction
certificate are traded and each certificate is equal to 1 tonne of CO2 reduction. One crucial
pre-condition of CDM is that the new projects should be in addition to those which were
already planned, so as to promote genuine new projects only which aim at reducing
emission. This is known as additionality clause. This mechanism on the one hand helps
Annex 2 countries in meeting their targets; it also helps developing countries in adopting
new technologies which are environmentally more sustainable.
Apart from these, there were also other flexible mechanisms to help Annex 2 countries in
meeting their targets i.e. International Emission Trading (IET) and Joint Implementation (JI).
Out of all these, CDM has been the most successful.
Kyoto Protocol- Legal but not enforceable, though Kyoto was legal, it didnt have provision
of enforcing penalties, as a result developed countries failed to meet the target and also got
away with that.
Doubts: ques7- It is not enforceable? Isn't it is binding on developed nation? Moreover
there are penalties for breaching of protocol by losing credit gaining privilege-ban in cap and
trade program including make up the difference in emission cap plus 30%?
Clarifications:
International environmental laws are generally initiated as soft laws that are not
enforceable per se but play a vital role in formulation of international environmental rulemaking.

www.iasbaba.com

46

Environment-Ecology-Biodiversity- Subject wise compilation of 60 Day Plan- 2016


Soft law sources point to the likely future direction of formally binding obligations by
indicating acceptable norms of behaviour.
They also facilitate codification and progressive development of rules of customary
international environmental law, which forms treaties and agreements at bilateral and
multilateral levels.
Sometimes, treaties and agreements remain at the soft law stage (for example Kyoto
Protocol) as a valuable instrument for enhancing or supplementing rules of international
environmental law within the treaties and agreements.
Though Kyoto was legal, it didnt have provision of enforcing penalties, as a result developed
countries failed to meet the target and also got away with that.
Yes, they do have enforcement branch but this does not mean they strictly enforce the
rules. It is upon States to make provisions or laws that makes them legally binding and
(especially) enforceable.

Q.91) Consider the following w.r.t the achievements of National Mission on Enhanced
Energy Efficiency
1. Implementation of the Compact Fluorescent Lamp Programme.
2. Green technologies Fiscal support, innovation, easier financing for green technologies.
3. Encouraging super critical thermal plants
4. Development in technology for Carbon Capture and Storage (CCS)
Select the correct code
a) 1, 2 and 3
b) 2, 3 and 4
c) 1, 2 and 4
d) All
Q.91) Solution (d)
Under mission National Mission on Enhanced Energy Efficiency under NAPCC, following
achievements have been made
Commencement of the first commitment period of PAT (Perform Achieve and Trade).
Expansion of the energy efficiency financing platform through memorandums of
understanding (MoU) with public-sector banks.
Implementation of the Compact Fluorescent Lamp Programme.
Green technologies Fiscal support, innovation, easier financing for green technologies.
Encouraging super critical thermal plants
Development in technology for Carbon Capture and Storage (CCS)
Q.92) Consider the following statements:
1. Clean Energy Cess is a kind of Carbon tax levied in India as a duty of excise on coal,
Lignite and peat.

www.iasbaba.com

47

Environment-Ecology-Biodiversity- Subject wise compilation of 60 Day Plan- 2016


2. In Budget of 2016-17, the Cess has been increased to Rs 100 per tonne on coal
production.
3. The Clean Energy Cess has been renamed as Swatch Bharat Cess.
Which of the above statements are incorrect?
a) 1 only
b) 2 and 3
c) 1 and 2
d) None of the above
Q.92) Solution (b)
Clean Energy Cess is a kind of carbon tax and is levied in India as a duty of Excise under
section 83 (3) of the Finance Act, 2010 on Coal, Lignite and Peat (goods specified in the
Tenth Schedule to the Finance Act, 2010) in order to finance and promote clean
environment initiatives, funding research in the area of clean environment or for any such
related purposes.
The Clean Energy Cess has been renamed to Clean Environment Cess and it is increased to
Rs400 per tonne on coal production.
Q.93) Which of the following statements are correct about a Community in terms of
levels of organization in ecology?
1. A community is the name given to a group of organisms of a single species.
2. Communities in most instances are named after dominant plant species.
Which of the above statements are correct?
a) 1 only
b) 2 only
c) Both 1 and 2
d) Neither 1 nor 2
Q.93) Solution (b)
Community is a name given to a group of species dependent on each other and living in one
ecosystem.
Communities in most instances are named after the dominant plant forms. Eg. Grassland is
named after grass, the most dominant specie. Although there are other species in grassland.
Q.94) Nitrogen gas forms 78% of the earths environment. Which of the following
statements are correct about atmospheric Nitrogen?
1. It creates an inert environment in atmosphere. If Nitrogen is not present, Oxygen would
react violently and oxidize(burn) carbon and other elements.
2. Nitrogen adds mass to air and helps in maintaining atmospheric pressure.

www.iasbaba.com

48

Environment-Ecology-Biodiversity- Subject wise compilation of 60 Day Plan- 2016


3. Nitrogen is taken up by humans through respiration from the atmosphere which helps in
building of amino acids and proteins.
4. Some plants have the ability to fix atmospheric nitrogen with the help of bacteria.
Select the code from below:
a) 1,2 and 3
b) 2,3 and 4
c) 1,2 and 4
d) All of the above
Q.94) Solution (c)
Humans do not use atmospheric nitrogen directly. Nitrogen is taken by body through food
materials.
Q.95) Consider the following statements:
1. Phagocytosis, process by which certain living cells called phagocytes ingest or engulf
other cells or particles.
2. The phagocyte may be a free-living one-celled organism, such as an amoeba, or one of
the body cells, such as a white blood cell.
Which of the above statements are correct?
a) 1 only
b) 2 only
c) Both 1 and 2
d) Neither 1 nor 2
Q.96) Solution (c)
Phagosytosis is the process by which a celloften a phagocyte or a protistengulfs a solid
particle to form an internal vesicle known as a phagosome.
It is a heterotrophic process of food intake.
Q.97) Which of the following statements are correct about the Niche of the species?
1. A niche is a unique functional role or place of a specie in an ecosystem.
2. It is a description of all the biological, physical and chemical factors that specie needs to
survive, reproduce and stay healthy.
3. Two or more species can have exact identical niches.
Select the code from below:
a) 1 and 2
b) 2 and 3
c) 1 and 3

www.iasbaba.com

49

Environment-Ecology-Biodiversity- Subject wise compilation of 60 Day Plan- 2016


d) All of the above
Q.97) Solution (a)
No two species can have an exact identical niche. (Refer Shankar IAS Environment)
Q.98) Which of the following statements are incorrect about ecotones?
1. It is a transition zone between two ecosystems with characteristics of adjacent
ecosystems.
2. Well-developed ecotones can have entirely different species from that of adjoining
communities.
3. Sometimes the number of species and the population density of some of the species is
much less in this zone than either community. This is called the edge effect.
Select the code from following:
a) 1 and 2
b) 3 only
c) 2 and 3
d) None of the above
Q.98) Solution (b)
Sometimes the number of species and the population density of some of the species is
much greater in this zone than either community. This is called the edge effect.
Q.99) Consider the following statements:
1. Aquatic ecosystems are not called Biomes.
2. Lotic water systems are fresh water ecosystems with stagnant water like pools, ponds,
lakes etc.
3. Lentic water bodies are fresh water ecosystems with moving water like streams, rivers,
waterfalls etc.
Which of the above statements are incorrect?
a) 1 and 2
b) 2 and 3
c) 1 only
d) None of the above
Q.99) Solution (b)
Fresh water systems are classified as lotic and lentic.
Lotic- moving water; Lentic- Stagnant water
Only terrestrial ecosystems are classified as biomes.

www.iasbaba.com

50

Environment-Ecology-Biodiversity- Subject wise compilation of 60 Day Plan- 2016


Q.100) Consider the following statements regarding estuaries:
1. Estuaries are located at the river mouth, where river meets the sea.
2. Estuary has very little wave action. So it provides a calm refuge to sea and also shelter to
many aquatic organisms.
3. It is the most productive region as it receives nutrients both from the rivers and the sea.
4. Predators are absent in estuaries.
Which of the above statements are correct?
a) 1,2 and 3
b) 2,3 and 4
c) 1,3 and 4
d) All of the above
Q.100) Solution (a)
Predators are an important part of estuary ecosystem. Where ever the conditions are
productive predators need to be there to check the population. They are the top most
consumer of the food chain.
Q.101) Bioremediation is the use of microorganisms to degrade the environmental
contaminants into less harmful forms. Consider the following about In-Situ
Bioremediation:
1. Bioventing: Supplying air through well into contaminated soil to stimulate the growth of
indigenous bacteria. It is used for simple hydrocarbons and can be used when
contamination is deep under surface.
2. Biosparging: injection of air under pressure below the water table to increase the
ground water oxygen concentration and enhance the rate of biological degradation of
contaminants through indigenous bacteria.
3. Bioaugmentation: microorganisms are imported to a contaminated site to enhance
degradation.
Which of the above terms are correctly defined?
a) 1 and 2
b) 3 only
c) 2 and 3
d) All of the above
Q.101) Solution (d)
Self Explanatory. Refer Shankar IAS chapter 5- Environmental Pollution.
Q.102) Consider the following attributes of Mangroves
1. It is a glycophypte

www.iasbaba.com

51

Environment-Ecology-Biodiversity- Subject wise compilation of 60 Day Plan- 2016


2. It has aerial roots
3. Buffer zone between land and sea
4. West Bengal has the highest mangrove cover in India
Select the correct code
a) 1,2 and 3
b) 1,2 and 4
c) 2,3 and 4
d) 1,3 and 4
Q.102) Solution (c)
It is a halophypte- Mangroves are salt tolerant trees, also called halophytes.
It has Aerial roots - Pneumatophore
Mangroves are buffers between the land and the sea. Coastlines throughout the world are
facing serious problems of coastal erosion and threat of rising sea levels due to global
warming have increased the threats by several folds. To control such assault of the sea on
land the nature has provided what is called as Mangroves, a tropical littoral ecosystem
which is more dynamic than the sea itself.
Rank
1
2
3
4
5
6
7
8
9
10

States/UTs with Highest


Mangrove Cover 2013
West Bengal
Gujarat
Andaman And Nicobar
Islands
Andhra Pradesh
Odisha
Maharashtra
Tamil Nadu
Goa
Kerala
Karnataka

Total Mangrove Cover in


km2
2,097
1,103
604
352
213
186
39
22
6
3

Q.103) What are the factors that affect the Rate of Decomposition?
1. Social Activity or Environment
2. Oxygen Level
3. Moisture Level
4. Soil pH
5. Temperature

www.iasbaba.com

52

Environment-Ecology-Biodiversity- Subject wise compilation of 60 Day Plan- 2016


Select the correct code
a) 1, 2, 4 and 5
b) 1, 2, 3 and 5
c) 2, 3, 4 and 5
d) 1, 3, 4 and 5
Q.103) Solution (c)
1. The most obvious cause of the break-down of soil organic matter is a high level of
biological activity and not social activity or environment.
2. Soil with a loose structure allows for ample spaces between soil particles for oxygen to
collect. In such soils, organic matter will decompose faster.
3. The amount of water in the soil, both indirectly and directly, affects the decomposition
rate of organic matter. Indirectly, a wet soil results in a slower break down because
water fills the air spaces in the soil, depriving the microbes of oxygen.
4. Low pH indicates an acidic soil, and this can have a major impact on the decomposition
of organic matter. Bacteria -- the organisms most responsible for breaking down organic
matter -- experience a sharp drop-off in activity once the pH drops below 6.0.
5. Soil organic matter breaks down faster in the summer, and areas with year-round warm
weather will decompose more organic matter annually than areas that experience cold
winters.
Q.104) Which of the following terms defines a transition between two biological
communities?
a) Ecotone
b) Ecotope
c) Ecological Backlash
d) Ecoline
Q.104) Solution (a)
Ecotone is a region of transition between two biological communities.
Ecoline is the rate of genetic change that occurs in an environment due to the merging of
different varieties of a plant species.
Q.105) Consider the following statements regarding Pygmy Hog:
1. They are the rarest and smallest breed of Pigs.
2. It is a critically endangered species found in tropical evergreen forests of western ghats.
3. Pygmy Hog sucking louse, a critically endangered species of insect, is a parasite found
only on Pygmy hog. Extinction of Pygmy hog will lead to its extinction too.
Which of the above statements are correct?
a) 1 and 2

www.iasbaba.com

53

Environment-Ecology-Biodiversity- Subject wise compilation of 60 Day Plan- 2016


b) 2 and 3
c) 1 and 3
d) All of the above
Q.105) Solution (c)
Pygmy hog is found in the grasslands of Assam.
Q.106) The Ramsar Convention is an international treaty for the conservation and
sustainable use of wetlands. Which of the following statements are correct about Ramsar
Convention?
1. The convention was signed in 1971 in the city of Ramsar, Iran.
2. Only natural wetlands are included under Ramsar Convention.
3. Conference of Parties (CoP), is the Conventions Governing body consisting of all
governments that have ratified the treaty.
Select the code from below:
a) 1 and 2
b) 2 and 3
c) 1 and 3
d) All of the above
Q.106) Solution (c)
Both natural and man-made wetlands are included in Ramsar Convention.
Q.107) There are two types of Biogeochemical cycles: the gaseous and the sedimentary
cycles. Consider the following statements:
1. In the gaseous cycle, the main reservoir of nutrients is the atmosphere and the ocean.
2. In the sedimentary cycle, the main reservoir is the soil and the sedimentary and other
rocks of the earths crust.
Which of the above statement(s) is/are correct?
a) 1 only
b) 2 only
c) Both 1 and 2
d) Neither 1 nor 2
Q.107) Solution (c)
There are two types of biogeochemical cycles : the gaseous and the sedimentary cycle.
In the gaseous cycle, the main reservoir of nutrients is the atmosphere and the ocean. In the
sedimentary cycle, the main reservoir is the soil and the sedimentary and other rocks of the
earths crust. (NCERT XI chapter 15)

www.iasbaba.com

54

Environment-Ecology-Biodiversity- Subject wise compilation of 60 Day Plan- 2016

Q.108) Consider the following statements regarding Oxygen Cycle:


1. Oxygen is the main by-product of respiration process.
2. Oxygen is the most abundant element in the earths crust.
Which of the above statement(s) is/are incorrect?
a) 1 only
b) 2 only
c) Both 1 and 2
d) Neither 1 nor 2
Q.108) Solution (a)
Oxygen is the main by-product of photosynthesis.
Q.109) Consider the following statements:
1. Nitrogen can be fixed in the atmosphere by lightning and cosmic radiation.
2. Denitrification is the process of conversion of nitrogen into nitrates.
Which of the above statement(s) is/are correct?
a) 1 only
b) 2 only
c) Both 1 and 2
d) Neither 1 nor 2
Q.109) Solution (a)
Denitrification is the process of conversion of nitrates into nitrogen.
Q.110) Which of the following are the type of temperate grasslands?
1. Prairies
2. Steppe
3. Savanna
4. Velds
Select the code from below:
a) 1 and 2
b) 2 and 3
c) 1,2 and 4
d) 3 and 4
Q.110) Solution (c)
Savanna are tropical grasslands.

www.iasbaba.com

55

Environment-Ecology-Biodiversity- Subject wise compilation of 60 Day Plan- 2016


*Question is corrected (It was wrongly given Velds as tropical grasslands)
Q.111) Consider the following statements:
1. Genetic diversity exists within species only.
2. Naturally interbreeding can occur within species.
Select the correct statement(s) from above:
a) 1 only
b) 2 only
c) Both 1 and 2
d) Neither 1 and 2
Q.111) Solution (b)
Genetic diversity exists within species as well as between species.
Q.112) Which of the following has a negative effect on agro-biodiversity?
1. GM crops
2. Fertilizers
3. Organic farming
4. Polyculture
Select the code from below:
a) 1 only
b) 1,2 and 3
c) 1 and 4
d) 2,3 and 4
Q.112) Solution (a)
GM crops have an invasive effect which eliminates the natural varieties of crops.
Rotation of crops or growing multiple crops in the same field is called Polyculture.
Q.113) UNESCOs Man and the Biosphere Programme (MAB) is an Intergovernmental
Scientific Programme that aims to establish a scientific basis for the improvement of
relationships between people and their environments. Which of following Indian
Biosphere reserves comes under the category of MAB?
1. Nilgiri
2. Gulf of Mannar
3. Agasthyamala
4. Rann of Kutch
Select the code from below

www.iasbaba.com

56

Environment-Ecology-Biodiversity- Subject wise compilation of 60 Day Plan- 2016


a)
b)
c)
d)

1 and 2
1,2 and 3
2,3 and 4
All of the above

Q.113) Solution (b)


Ten of the eighteen biosphere reserves are a part of the World Network of Biosphere
Reserves, based on the UNESCO Man and the Biosphere (MAB) Programme list:
Nilgiri Biosphere Reserve
Gulf of Mannar Biosphere Reserve
Sundarbans Biosphere Reserve
Nanda Devi Biosphere Reserve
Nokrek Biosphere Reserve
Pachmarhi Biosphere Reserve
Simlipal Biosphere Reserve
Achanakmar-Amarkantak Biosphere Reserve
Great Nicobar
Agasthyamala Biosphere Reserve
Q.114) Dirty Dozen refers to:
1. 12 most harmful Greenhouse Gases
2. 12 Ozone Depleting Substances
3. 12 Persistent Organic Pollutants
4. 12 carcinogenic metals
Q.114) Solution (c)
These were the 12 initial compounds that were listed under the Stockholm convention.
Q.115) Why biodiversity is rich in Tropics compared to Temperate?
1) Tropics have had more stable climate
2) Warm temperatures and high humidity in most tropical areas
3) Tropical communities are older than temperate ones
4) Greater pressure from pests, parasites and diseases in tropics
Which of the statements given above is/are correct reasons for the same?
a) 1 and 2 only
b) 2, 3 and 4 only
c) 1, 2 and 3 only
d) All of the above
Q.115) Solution (d)
www.iasbaba.com

57

Environment-Ecology-Biodiversity- Subject wise compilation of 60 Day Plan- 2016


Explanation:
Reasons why biodiversity is rich in Tropics:
1. Over geological times the tropics have had more stable climate than the temperate
zones. In tropics, therefore, local species continued to live there itself, whereas in
temperate they tend to disperse to other areas.
2. Tropical communities are older than temperate ones and therefore there has been more
time for them to evolve. This could have allowed them greater degree of specialization
and local adaptation to occur.
3. Warm temperatures and high humidity in most tropical areas provide favourable
conditions for many species that are unable to survive in temperate areas.
4. In tropics there may be greater pressure from pests, parasites and diseases. This does
not allow any single species to dominate and thus there is opportunity for many species
to co-exist. On the contrary in temperate zones there is reduced pests pressure due to
cold, and there is one or few dominating species that exclude many other species.
5. Among plant, rates of out-crossing appear to be higher in tropics, which may lead to
higher levels of genetic variability.
6. Tropical areas receive more solar energy over the year. Thus tropical communities are
more productive or greater resource base that can support a wider range of species.
Q.116) Which among the following statement/s is/are correct in regard to National Parks,
Wildlife Sancturies and Biosphere Reserves?
1) The rights of the people living inside the National parks and Wildlife Sanctuaries are
tightly regulated
2) National parks provide outdoor recreation, camping opportunities and are designed to
educate the public on the importance of conservation activities
3) Tourism is normally not permissible in Biosphere Reserve
4) In National Parks and Biosphere Reserves, boundaries are fixed by legislation. However,
boundaries of sancturies are not sacrosanct
Choose the appropriate code:
a) 1 and 2 only
b) 2, 3 and 4 only
c) 1, 2 and 3 only
d) All of the above
Q.116) Solution (b)
Explanation:
A National Park is an area which is strictly reserved for the betterment of the wildlife
and where human activities like forestry, grazing or cultivation are not permitted.

www.iasbaba.com

58

Environment-Ecology-Biodiversity- Subject wise compilation of 60 Day Plan- 2016

A Wildlife Sanctuary is a protected area that is reserved for the conservation only of
wildlife animals and plant species. Human activities like harvesting or timber,
collection of minor forest products and private ownership rights are allowed.

Doubts:
Babaji.....Que.
No.
12
When
talking
about
Biosphere
Reserves http://www.moef.nic.in/divisio... Site says...Tourism is permitted in Buffer
Zone..and your question explicitly says that tourism is not permitted in BIOSPHERE
RESERVE....
Why do you make the question so ambiguous???
Clarification:
Read the question carefully. If you can eliminate 1st statement, directly you get the answer.
And what is ambiguous?? Statement (3)??
Tourism is normally not permissible in Biosphere Reserve
Read the below (excerpt from http://www.moef.nic.in/division/biosphere-reserves)
Biosphere reserves are demarcated into following 3 inter-related zones:
Core Zone
Core zone must contain suitable habitat for numerous plant and animal species, including
higher order predators and may contain centres of endemism. Core areas often conserve
the wild relatives of economic species and also represent important genetic reservoirs
www.iasbaba.com

59

Environment-Ecology-Biodiversity- Subject wise compilation of 60 Day Plan- 2016


having exceptional scientific interest. A core zone being National Park or
Sanctuary/protected/regulated mostly under the Wildlife (Protection) Act, 1972. Whilst
realizing that perturbation is an ingredient of ecosystem functioning, the core zone is to be
kept free from human pressures external to the system.
Buffer Zone
The buffer zone, adjoins or surrounds core zone, uses and activities are managed in this
area in the ways that help in protection of core zone in its natural condition. These uses
and activities include restoration, demonstration sites for enhancing value addition to the
resources, limited recreation, tourism, fishing, grazing, etc; which are permitted to reduce
its effect on core zone. Research and educational activities are to be encouraged. Human
activities, if natural within BR, are likely to continue if these do not adversely affect the
ecological diversity.
What do you understand by this?? Tourism is allowed but there are some conditions or
limitations. It should see that it helps in protection of core zone in its natural condition and
should not affect the core zone.
So we can normally say - Tourism is normally not permissible in Biosphere Reserve
And this statement is no way explicitly saying that tourism is not permitted in BIOSPHERE
RESERVE (which you have misunderstood)
For more info,

Refer Class 10 Environmental Education, ICSE by P.S.Latika (Chap 6, page 51)

Read this link:

http://www.thehindu.com/sci-tech/question-corner/article7252050.ece#comments

Q.117) Which organisation released the recent report Global Urban Ambient Air Pollution
Database 2016?
a) World Health Organization
b) United Nations Environment Programme (UNEP)
c) United Nations Development Programme
d) Mdecins Sans Frontires
Q.117) Solution (a)
Explanation:
1. WHO released report Global Urban Ambient Air Pollution Database (update 2016)
2. The Report says Delhi is no longer the most polluted city in the world. Delhi now stands
11th among 3,000 cities in 103 countries (in terms of fine particulate matter or PM 2.5)
3. Four Indian cities are among the worlds 10 most polluted
4. Delhis place as the most polluted is taken by Zabol, in Iran.

www.iasbaba.com

60

Environment-Ecology-Biodiversity- Subject wise compilation of 60 Day Plan- 2016


5. Gwalior and Allahabad, meanwhile, come a close second and third in terms of PM 2.5,
while Patna and Raipur are ranked 6th and 7th.
Q.118) Consider the following statements:
1) A biodiversity hotspot is a region with a high number of endemic species.
2) The concept of biodiversity hotspots was originated by Norman Myers.
3) Hotspots are formally recognised or governed areas.
Which of the statements given above are correct?
a) Only 1 and 2
b) Only 2 and 3
c) Only 1 and 3
d) All of the above
Q.118) Solution (a)
Explanation:
Statement 1 and 2 are correct
Hotspots are not formally recognised or governed areas. However, the identification of
these areas as hotspots increases the likelihood of conservation investment.
In addition, other designations for biodiversity conservation are likely to be present
within these broad areas which may have more formal management structures.
For example, the average protected area coverage of hotspots, based on IUCN protected
area categories I-VI, is 10% of their original extent.
Q.119) Which lake is the biggest for Brackish Water (Saline) lagoon, it is set to be Indias
biggest and world second biggest coastal lagoon?
a) Lunar Lake, Maharashtra
b) Sambhar Lake, Rajasthan
c) Chilika Lake, Orissa
d) Kuttanad lagoon, Kerala
Q.119) Solution (c)
Explanation:
1. Chilika lake of Odisha is the biggest for Brackish Water (Saline) lagoon, it is set to be
Indias biggest and world second biggest coastal lagoon.
2. Brackish seas: Baltic Sea (the worlds largest pool of brackish water); Black Sea
3. Brackish water lakes: Caspian Sea (worlds largest lake); Lake Charles in Lake Charles,
Louisiana, the United States; Chilika Lake, Odisha, India.
Q.120) Consider the following statements with regard to Snow Leapord, classified as
Endangered by the IUCN, which was recently in news:

www.iasbaba.com

61

Environment-Ecology-Biodiversity- Subject wise compilation of 60 Day Plan- 2016


a) The snow leopard is a large cat native to the mountain ranges of Central and South Asia.
b) Snow leopards inhabit alpine and subalpine zones at elevations from 3,000 to 4,500 m
(9,800 to 14,800 ft).
c) It is found in Jammu and Kashmir, Uttarakhand, Himachal Pradesh, Sikkim, West Bengal
and Arunachal Pradesh.
Choose the appropriate code from below:
a) 1 only
b) 2 only
c) 1 and 2 only
d) All of the above
Q.120) Solution (c)
Explanation:
The snow leopard is a large cat native to the mountain ranges of Central and South Asia.
Snow leopards inhabit alpine and subalpine zones at elevations from 3,000 to 4,500 m
(9,800 to 14,800 ft). In the northern range countries, they also occur at lower elevations.
Jammu and Kashmir, Uttarakhand, Himachal Pradesh, Sikkim, and Arunachal Pradesh
(not found in West Bengal)
Q.121) When a fossil fuel burns, two of the gases that contribute to acid rain are released.
These two gases are:
a) methane and carbon dioxide.
b) nitrogen and sulfur dioxide.
c) methane and sulfur dioxide.
d) carbon dioxide and sulfur dioxide.
Q.121) Solution (d)
Explanation:
Factual question self explanatory
Doubts:
Q.17 option should be B,..https://en.wikipedia.org/wiki/..
Can anybody explain Q.17? Acid Rain is caused by NOx and SOx
Clarification:
Absolutely correct if ques asks about acid rain
But Ques asks - "When a fossil fuel burns", what gases are released
Explanation:

www.iasbaba.com

62

Environment-Ecology-Biodiversity- Subject wise compilation of 60 Day Plan- 2016


While you consider this condition, given answer is correctThe fossil fuels are coal, oil and
natural gas. They are fuels because they release heat energy when they are burned. They
are fossil fuels because they were formed from the remains of living organisms millions of
years ago.
Fossil fuels release carbon dioxide when they burn, which adds to the greenhouse effect
and increases global warming. Of the three fossil fuels, for a given amount of energy
released, coal produces the most carbon dioxide and natural gas produces the least.
Coal and oil release sulfur dioxide gas when they burn, which causes breathing problems
for living creatures and contributes to acid rain.
So, majorly Fossil fuels produce carbon-dioxide and sulfur dioxide
Link:
http://www.bbc.co.uk/schools/gcsebitesize/science/aqa_pre_2011/energy/mainselectric
ityrev1.shtml

Q.122) Consider the following statements about Basel Convention:


1) It is an international treaty that was designed to reduce the movements of hazardous
waste between nations.
2) It specifically focuses on to prevent transfer of hazardous waste from developed to less
developed countries (LDCs).
3) It also addresses the movement of radioactive waste between different nations.
Select the correct code:
a) Only 1 and 2
b) Only 2 and 3
c) Only 1 and 3
d) All of the above
Q.122) Solution (a)
Explanation:
Basel Convention is an international treaty that was designed to reduce the movements
of hazardous waste between nations
It specifically focuses on to prevent transfer of hazardous waste from developed to less
developed countries (LDCs)
It does not, however, address the movement of radioactive waste
Link: http://www.basel.int/portals/4/basel%20convention/docs/text/baselconventiontexte.pdf
Q.123) Consider the following statements about World Wide Fund for Nature (WWF)

www.iasbaba.com

63

Environment-Ecology-Biodiversity- Subject wise compilation of 60 Day Plan- 2016


1) WWF is an international non-governmental organization
2) WWF headquarter is located in Switzerland
3) The groups mission is to stop the degradation of the planets natural environment and
to build a future in which humans live in harmony with nature
Choose the correct code
a) 1 only
b) 1 and 3 only
c) 1 and 2 only
d) All of the above
Q.123) Solution (d)
Explanation:
All the statements are correct and self explanatory
WWF is an international non-governmental organization, headquarter is located in
Switzerland
The groups mission is to stop the degradation of the planets natural environment and
to build a future in which humans live in harmony with nature.
Q.124) Which of the following is/are not an ingredient of Smog?
1) Nitrogen oxides
2) Hydrocarbons
3) Polycarbonates
4) Ozone
Select the correct answer using the code given below:
a) 1 and 4 only
b) 1 and 3 only
c) 3 only
d) None of the above
Q.124) Solution (c)
Explanation:
The word SMOG is a combination of the words SMOke" and "fOG". SMOG is the haze that
can be seen over busy cities and other areas with a lot of automobile traffic or industrial
factories. SMOG is especially bad when the temperature rises in the summer.
SMOG is made up of many gases and small bits of "STUFF". The most common SMOG
ingredients include:
nitrogen oxides
sulphur oxides
oxidized organic compounds

www.iasbaba.com

64

Environment-Ecology-Biodiversity- Subject wise compilation of 60 Day Plan- 2016

aldehydes
volatile organic compounds (VOC's)

The most common cause of SMOG is from the reaction of nitrogen oxides with
hydrocarbons in the presence of sunlight. This reaction produces OZONE.

Q.125) Which of the following two criteria have to be met in order to qualify as a
biodiversity hotspot on the world hotspots map?
1. The region must contain at least 0.5 percent or 1500 species of vascular plants as
endemic species.
2. The region has to have lost at least 70 percent of its primary vegetation.
3. The region must not have lost any of its primary predators.
Choose the correct option:
a) 1 and 2
b) 2 and 3
c) 1 and 3
d) All of the above
Q.125) Solution (a)
A biodiversity hotspot is a bio-geographic region with a significant reservoir of biodiversity
that is under threat from humans. The concept of biodiversity hotspots was originated by
Norman Myers in two articles in The Environmentalist. To qualify as a biodiversity hotspot
on Myers 2000 edition of the hotspot-map, a region must meet two strict criteria: it must
contain at least 0.5% or 1,500 species of vascular plants as endemics, and it has to have lost
at least 70% of its primary vegetation.
Q.126) Which of the following are included in the Red Data book of rare and endangered
species?
1. Plants
2. Animals
3. Fungi

www.iasbaba.com

65

Environment-Ecology-Biodiversity- Subject wise compilation of 60 Day Plan- 2016


Select the code from the following:
a) 1 and 2 only
b) 2 only
c) 1 and 3
d) All of the above
Q.126) Solution (d)
A Red Data Book contains lists of species whose continued existence is threatened. Species
are classified into different categories of perceived risk. Each Red Data Book usually deals
with a specific group of animals or plants and fungi. They are now being published in many
different countries and provide useful information on the threat status of the species.
Q.127) Consider the following statements regarding Kyoto Protocol:
1. It is binding for Annex I countries.
2. The targets initially applied for six Green house Gases which were translated into CO2
equivalents in determining reductions in emissions.
3. The signatories have to reduce their individual Green House Gas emissions by 5.2% from
1990 level.
Which of the above statements are correct?
a) 1 and 2
b) 2 and 3
c) 1 and 3
d) All of the above
Q.127) Solution (a)
Kyoto Protocol of COP-3 to the UNFCCC is a legally binding agreement for Annex-I countries
to reduce their collective Green House Gas emissions by 5.2% from 1990 level. The
reduction obligation under Kyoto Protocol is additional to the CFCs and industrial gas
emissions covered under the Montreal Protocol.
Q.128) Consider the following statements:
1. Ozone is generally formed in upper stratosphere and sinks to lower stratosphere.
2. Ozone is measured in Dobson Unit in stratosphere.
Which of the above statements are correct?
a) 1 only
b) 2 only
c) Both 1 and 2
d) Neither 1 nor 2

www.iasbaba.com

66

Environment-Ecology-Biodiversity- Subject wise compilation of 60 Day Plan- 2016


Q.128) Solution (c)
The upper stratosphere is also known as Chemosphere because of the chemical reactions
taking place there.
A dobson unit is the most basic measure used in ozone research. One Dobson Unit (DU) is
defined to be 0.01 mm thickness at STP (standard temperature and pressure).
Q.129) Which among the following awards has been institued by the Government of India
for individuals or communities from rural areas that have shown extraordinary courage
and dedication in protecting Wildlife?
a) Indira Gandhi Paryavaran Puraskar
b) Medini Puruskar Yojana
c) Amrita Devi Bishnoi Award
d) Pitambar Pant National Award
Q.129) Solution (c)
This award is given for significant contribution in the field of wildlife protection, which is
recognised as having shown exemplary courage or having done exemplary work for the
protection of wildlife. A cash award of Rupees One lakh is presented to
individuals/institutions involved in wildlife protection.
Q.130) Consider the following statements regarding Dark Fermentation:
1. It is a method to produce Methane from solid Organic waste.
2. Dark fermentation differs from photofermentation in that it proceeds without the
presence of light.
Which of the above statements are correct?
a) 1 only
b) 2 only
c) Both 1 and 2
d) Neither 1 nor 2
Q.130) Solution (b)
It is a method to produce Hydrogen as fuel from wastewater.
Q.131) Consider the following statements regarding Carbon Monoxide:
1. CO is naturally formed in human body.
2. CO is a normal neurotransmitter.
3. Too much of Carbon monoxide is poisonous for humans.
Which of the above statements are incorrect?
a) 1 and 2

www.iasbaba.com

67

Environment-Ecology-Biodiversity- Subject wise compilation of 60 Day Plan- 2016


b) 2 and 3
c) 2 only
d) None of the above
Q.131) Solution (d)
None of the above statement is incorrect.
Carbon monoxide is naturally produced by the action of heme oxygenase 1 and 2 on the
heme from hemoglobin breakdown. This process produces a certain amount of
carboxyhemoglobin in normal persons, even if they do not breathe any carbon monoxide.
Following the first report that carbon monoxide is a normal neurotransmitter in 1993, as
well as one of three gases that naturally modulate inflammatory responses in the body (the
other two being nitric oxide and hydrogen sulfide), carbon monoxide has received a great
deal of clinical attention as a biological regulator. In many tissues, all three gases are known
to act as anti-inflammatories, vasodilators, and promoters of neovascular growth. Clinical
trials of small amounts of carbon monoxide as a drug are ongoing. Nonetheless, too much
carbon monoxide causes carbon monoxide poisoning.
Q.132) Emu farms are being promoted in India. Which of the following statements are
correct about Emu?
1. It is one of the heaviest flying birds with Australian origin.
2. Emu meat has much lower fat compared to chicken, mutton, pork or turkey, & low in
cholesterol, hence good for heart patients.
3. Emu egg is a great nutritional supplement as one egg can feed an entire family.
Select the code from following:
a) 1 only
b) 2 and 3
c) 1 and 2
d) All of the above
Q.132) Solution (b)
Emu is flightless bird of Australian Origin.
http://agrifarming.in/emu-farming-profits
Q.133) To increase yield, the cows are injected with Oxytocin, a hormone banned in India
under the Prevention of Cruelty to Animals Act and section 12 of Food and Drug
Adulteration Prevention Act, 1960. Why is this hormone banned?
1. Milk produced from such cows is unfit/harmful for human consumption.
2. Cows injected with Oxytocin have a greater incidence of abortions, mastitis and lower
conception rates.

www.iasbaba.com

68

Environment-Ecology-Biodiversity- Subject wise compilation of 60 Day Plan- 2016


3. Calves born to Oxytocin injected cows suffer higher than normal infant mortality and
delayed puberty.
Select the code from below:
a) 1 only.
b) 1 and 2
c) 2 and 3
d) All of the above
Q.133) Solution (c)
Although research is going on, there is no conclusive report that it has negative impact on
human health.
Q.134) Rabies has caused a menace in India and it has been made a priority disease for
control under 12th 5 year plan. The bite of which of the following species causes rabies?
1. Dogs
2. Monkeys
3. Bats
Select the code from below:
a) 1 only
b) 1 and 3
c) 1 and 2
d) All of the above
Q.134) Solution (d)
Self explanatory.
Q.135) Consider the following statements in regard to Aquatic Ecosystem:
1) Neustons are unattached organisms which live at the air-water interface
2) Nektons are organisms which remain attached to stems and leaves of rooted plants and
substances emerging above the bottom mud
3) Planktons includes both microscopic plants like algae and animals like crustaceans and
protozoans
4) Benthos or benthic organisms are those found living in the bottom of the water mass
Which of the above given statement(s) is/are correct?
a) 3 and 4 only
b) 1, 2 and 4 only
c) 1, 3 and 4 only
d) All of the above

www.iasbaba.com

69

Environment-Ecology-Biodiversity- Subject wise compilation of 60 Day Plan- 2016

Q.135) Solution (c)


Explanation:
Neustons are unattached organisms which live at the air-water interface such as floating
plants
Periphytons are organisms which remain attached to stems and leaves of rooted plants
and substances emerging above the bottom mud such as sessile algae and their
associated group of animals
Planktons includes both microscopic plants like algae (phytoplanktons) and animals like
crustaceans and protozoans (zooplanktons) found in all aquatic ecosystems, except
certain swift moving waters.
Nektons are groups which contains animals which are swimmers
Benthos or benthic organisms are those found living in the bottom of the water mass
Q.136) Consider the following two statements:
1) The Centre can declare any animal 'vermin', under the Wildlife Protection Act, 1972,
following requests from the respective States.
2) Vermin provision can be utilized within specified territories of the States, and outside
forests and protected areas.
Which of the above given statement(s) is/are correct?
a) 1 only
b) 2 only
c) Both
d) None
Q.136) Solution (b)
Explanation:
As per Section 62 of the Wildlife Protection Act, 1972, States can send a list of wild
animals to the Centre requesting it to declare them vermin for selective slaughter. The
Central Government may by notification, declare any wild animal other than those
specified in Schedule I and part 11 of Schedule H of the law to be vermin for any area
for a given period of time. As long as the notification is in force such wild animal shall be
included in Schedule V of the law, depriving them of any protection under that law.
This reprieve means that those who kill these animals here will, for a year after these
notifications come into effect, not be subject to the jail terms and fines that hunting
these animals typically invite.
Do you know?
Wildlife laws divide species into schedules ranked from I to V. Schedule I members are the
best protected, in theory, with severe punishments meted out to those who hunt them.
Wild boars, nilgai and rhesus monkeys are Schedule II and III members also protected,
www.iasbaba.com

70

Environment-Ecology-Biodiversity- Subject wise compilation of 60 Day Plan- 2016


but can be hunted under specific conditions. Crows and fruit bat fall in Schedule 5, the
vermin category.
http://www.thehindu.com/opinion/op-ed/what-is-and-isntvermin/article8742606.ece?utm_source=InternalRef&utm_medium=relatedNews&utm_ca
mpaign=RelatedNews
Q.137) Which among the following ecological terms is/are not true?
1) An Ecad is a population of individuals, which are genetically different.
2) The place where an organism lives, eats and reproduces is known as its Niche.
Select the appropriate code:
a) 1 only
b) 2 only
c) Both
d) None
Q.137) Solution (c)
Explanation:
Statement (1) is definition of Ecotype and Statement (2) is of Habitat.

HabitatThe place where an organism lives, eats and reproduces is known as its
habitat. The habit of an organism actually represents a particular set of environmental
conditions suitable, for its successful growth.
A habitat is where an organism or a community of organisms lives, and a niche is the
specific place an organism has in an ecosystem. A habit can help define the niche of
particular creature but cannot describe it entirely.
Important terms:
EcadAn ecad is a population of individuals, which although belong to the same genetic
stock, but differ markedly in external characters such as size, shape and colour etc.
Ecological Niche and Ecological EquivalentEcological niche of an organism is the
physical space occupied by it, its functional role in the community i.e. tropic position, its
position in environment and the conditions of existence.
Organisms that occupy the same or similar ecological niches in different geographical
regions are known as Ecological Equivalents.
Ecosystem EcologyRelation and interaction of both plant and animal communities
with their total environment.
EcotypePopulation of individuals of a species, which are genetically different.
Environmental Biology or EcologyLiving organisms are inseparably related with their
physical and biological surroundings. This interrelationship of organisms with their
physical and biotic environments is studied under a separate discipline of science, which
is known as environmental biology or ecology.

www.iasbaba.com

71

Environment-Ecology-Biodiversity- Subject wise compilation of 60 Day Plan- 2016

Q.138) Which of the following are the physical processes responsible for the formation of
Estuaries?
1) Rising sea level
2) Movement of sand and sandbars
3) Glacial processes
4) Tectonic processes
Select the appropriate code:
a) All of the above
b) 1 and 4 only
c) 1, 3 and 4 only
d) 1, 2 and 4 only
Q.138) Solution (a)
Explanation:
Most estuaries can be grouped into four geomorphic categories based on the physical
processes responsible for their formation:
1) Rising sea level
2) Movement of sand and sandbars
3) Glacial processes
4) Tectonic processes
Q.139) Consider the following differences between Turtle and Tortoise:
1) Tortoises are land dwelling animals, while turtles are water-dwelling animals
2) Tortoises are found mostly in Asia and Africa, and some species in South Americas
Galpagos Islands, while turtles are mainly found in Africa and America
3) Tortoises are mainly herbivorous, carnivorous or omnivorous i.e. mostly are carnivorous
or omnivorous. Turtles are primarily herbivorous animals and few are omnivorous.
4) Turtles feet are short and sturdy with bent legs
Which of the above given difference(s) is/are not correct?
a) All of the above
b) 1 and 3 only
c) 3 and 4 only
d) 2 and 4 only
Q.139) Solution (c)
Explanation:

www.iasbaba.com

72

Environment-Ecology-Biodiversity- Subject wise compilation of 60 Day Plan- 2016

www.iasbaba.com

73

Environment-Ecology-Biodiversity- Subject wise compilation of 60 Day Plan- 2016

Q.140) Regions which receives rainfall from North-Easterly monsoon are:


1. Assam and Meghalaya
2. West Bengal
3. Tamil Nadu
4. Kerala
Select the code from the following:

www.iasbaba.com

74

Environment-Ecology-Biodiversity- Subject wise compilation of 60 Day Plan- 2016


a)
b)
c)
d)

1 and 2
3 only
1 and 4
1,2 and 4

Q.140) Solution (b)


North Easterly monsoons blows from North- East direction towards South-West. They cause
winter rainfall in Tamil Nadu Coast.
Q.141) Which of the following statements are not correct about the Lakshadweep islands?
1. They are all coral islands.
2. Rainfall is very scarce
3. They are the extended part of the continent.
4. They have Indias only active volcano, i.e. Barren Island.
Select the code from below:
a) 1,2 and 4
b) 1,2 and 3
c) 2,3 and 4
d) All of the above
Q.141) Solution (c)
Lakshadweep islands have a coral origin.
Barren island lies in Andaman and Nicobar islands.
Q.142) Consider the following statements:
1. Weather is the average atmospheric condition of an area over a considerable time.
2. Climate is a short term condition of temperature, pressure and humidity.
Which of the above statements are correct?
a) 1 only
b) 2 only
c) Both 1 and 2
d) Neither 1 nor 2
Q.142) Solution (d)
Climate is the average atmospheric condition of an area over a considerable time.
Q.143) Consider the following:
Nomads
Place
1. Bindibu
Australia

www.iasbaba.com

75

Environment-Ecology-Biodiversity- Subject wise compilation of 60 Day Plan- 2016


2. Bedouin
3. Tuaregs
4. Bushmen

Arabia
Sahara
Kalahari

Which of the above are correctly matched?


a) 2 only
b) 1,2 and 4
c) 2,3 and 4
d) All of the above
Q.143) Solution (d)
Self Explanatory
Q.144) Consider the following statements regarding Gobi Desert:
1. Gobi desert has a scarcity of water as snow which falls there evaporates directly through
sublimation.
2. Bactrian Camel survives in Gobi desert by directly eating ice/snow which most of the
other animals cant do.
Which of the above statements are correct?
a) 1 only
b) 2 only
c) Both 1 and 2
d) Neither 1 nor 2
Q.144) Solution (c)
Gobi desert is a cold desert in Mongolia. Precipitation mostly occurs in the form of snowfall.
Although the quantity of snowfall is good enough, the land does not get the water as it
evaporates directly through sublimating i.e. without getting converted into liquid water.
This is the reason why there is scarce vegetation.
Bactrian camel has adapted to this environment. They can directly eat snow and store
water.
Q.145) Consider the following statement regarding Mediterranean climatic region:
1. During summers, the trade winds are offshore and there is practically no rainfall.
2. They have open woodlands, of which Cork Oak trees are the best known. Cork oaks are
valued for their thick barks used for making corks of wine bottles.
3. Animal farming is an important activity because of soft nutritious grass.
Which of the above statements are correct?
a) 1 and 2

www.iasbaba.com

76

Environment-Ecology-Biodiversity- Subject wise compilation of 60 Day Plan- 2016


b) 2 and 3
c) 1 and 3
d) All of the above
Q.145) Solution (a)
During summers the ITCZ shifts northwards. The trade winds blow offshore the
Mediterranean region. Hence practically there is no summer rainfall in this region.
Conditions in Mediterranean do not suit grass, because most of the rain comes in the cool
season when growth is slow. Even if the grass survives they are wiry and bunchy that they
are not suitable for animal farming. Cattle rearing is thus unimportant in Mediterranean.
(G.C. Leong)
Q.146) Consider the following statement regarding Hangul:
1. Hangul is the only surviving member of the Asiatic Red Deer family.
2. It is critically endangered and found only in the forests of Jammu and Kashmir,
Uttarakhand and Sikkim.
3. It has been named as the state animal of Jammu and Kashmir.
Which of the above statements are incorrect?
a) 1 and 3
b) 2 only
c) 1 and 2
d) None of the above
Q.146) Solution (b)
Hangul or Kashmir stag is confined to the Dachigam National Park of Kashmir.
Doubts:
Hangul is not a member of "Red deer" family anymore. It belongs to Asian clade of Elk and it
is also not classified as critically endangered by IUCN but comes under not evaluated
category. And it is found in Northern reaches of Chamba Valley of H.P. as well.
Clarification:
Refer
to
this
article:
An
eye
on
the
Hangul
or
http://www.sanctuaryasia.com/magazines/conservation/10115-a-critical-note-on-thestatus-of-hangul-kashmir-deer.html

Q.147) Consider the following statements regarding the Black Lung Disease:
1. It is an industrial hazard of mercury mining and develops due to inhaling of mercury
vapours.
2. It is a form of Pneumoconiosis.

www.iasbaba.com

77

Environment-Ecology-Biodiversity- Subject wise compilation of 60 Day Plan- 2016


Which of the above statements are correct?
a) 1 only
b) 2 only
c) Both 1 and 2
d) Neither 1 nor 2
Q.147) Solution (b)
The other name for Black Lung Disease is Coal Workers Pneumoconiosis (CWP) and is
caused by long exposure to coal dust. It is similar to silicosis, from inhaling silica dust, and to
the long effects of Tobacco smoking.
Q.148) Consider the following statements:
1. Hypothetically, if we remove all the ozone from the stratosphere, the temperature of
stratosphere will increase.
2. Green house gases are responsible for ozone hole in stratosphere.
3. Ozone holes are found only over the Polar Regions.
Which of the above statements are incorrect?
a) 1 and 2
b) 2 and 3
c) 1 and 3
d) All of the above
Q.148) Solution (a)
Temperature of the stratosphere increases with increase in height because ultraviolet
radiations are absorbed by ozone. The radiations are helpful to carry forward the chemical
reactions in chemosphere. If the ozone layer is removed, the temperature of stratosphere
will decrease.
Ozone holes are caused by CFCs. Although CFCs are also green house gases but we cannot
have a sweeping remark that green house gases are responsible.
Q.149) Lichens are important in the studies on atmosphere pollution because they
a) Can grow in greatly polluted atmosphere
b) Can readily multiply in polluted atmosphere
c) Effectively purify the atmosphere
d) are very sensitive to pollutants
Q.149) Solution (d)
Lichens are highly sensitive to pollutants and a slight increase in pollution level can affect
the population of lichens.

www.iasbaba.com

78

Environment-Ecology-Biodiversity- Subject wise compilation of 60 Day Plan- 2016


Q.150) Which among the following is/are economic importance of Algae?
1) Algae as Food for Humans
2) Algae in Agriculture
3) Algae in Industry
4) Algae in space travel
Choose the appropriate code:
a) 2 and 3 only
b) 2, 3 and 4 only
c) 1, 2 and 3 only
d) All of the above
Q.150) Solution (d)
Algae as Food: Algae are important as a source of food for human beings, domestic animals
and fishes. Species of Porphyra are eaten in Japan, England and USA. Ulva, Laminaria,
Sargassum and Chlorella are also used as food in several countries. Sea weeds (Laminaria,
Fucus, Ascophyllum) are used as fodder for domestic animals.
Algae in Agriculture: Various blue green algae such as Oscillatoria, Anabaena, Nostoc,
Aulosira increase the soil fertility by fixing the atmospheric nitrogen. In view of the
increasing energy demands and rising costs of chemically making nitrogenous fertilizers,
much attention is now being given to nitrogen fixing bacteria and blue green algae. Many
species of sea weeds are used as fertilizers in China and Japan.
Algae in Industry: Agar agar is used in the preparations of some medicines and cosmetics.
It is obtained from the red algae Gelidium and Gracillaria.
A phycocolloid Alginic acid is obtained from brown algae. Algin is used as emulisifier in ice
creams, tooth pastes and cosmetics.
Idodine is obtained from kelps (brown algae) especially from speicies of Laminaria.
Algae in space travel: Chlorella pyrenoidosa is used in space travel to get rid of Co2 and
other body wastes. The algae multiplies rapidly and utilizes the Co2 and liberate 02 during
photosynthesis. It decomposes human urine and faeces to get N2 for protein synthesis.
Q.151) Which of following statement is incorrect about the Biosphere?
a) Biosphere is combination of lithosphere, hydrosphere and Atmosphere
b) Biosphere is missing at extreme of north and south pole
c) Organisms are uniformly present in Biosphere
d) None of the above
Q.151) Solution (c)
Biosphere is combination of lithosphere, hydrosphere and Atmosphere. Biosphere is not
found everywhere some areas do not support the conditions required for Biosphere. Hence
Organisms are not uniformly distributed throughout the world.

www.iasbaba.com

79

Environment-Ecology-Biodiversity- Subject wise compilation of 60 Day Plan- 2016

Q.152) Choose the Correct Answer


Hydroponics is otherwise called
a) soil-less agriculture
b) tank farming
c) chemical gardening
d) all the above
Q.152) Solution (d)
Hydroponics: The term hydroponics has been used for growth of plants in water and sand
culture. This may also be referred to as soil-less agriculture, test-tube farming, tank farming
or chemical gardening.
Q.153) The entire component of species of organism plants and animals, found within a
given region is called
a) Biota
b) Niche
c) Community
d) Habitat
Q.153) Solution (a)
The entire component of species of organism plants and animals, found within a given
region is known as Biota.
Doubts:
Community and biota is the same concept. please refer:http://www.biologydiscussion.c...
"..A community, technically often referred to as biota or biotic community is a local
association of several species populations. According to Krebs (1994), a community is an
assemblage of the populations of living organisms in a prescribed area or habitat..."
Clarification:
There is very less information in regard to definitions of Biota and Community.
A biota is the total collection of organisms of a geographic region or a time period, from
local geographic scales and instantaneous temporal scales all the way up to whole-planet
and whole-timescale spatiotemporal scales. The biotas of the Earth make up the biosphere.
In ecology, a community or biocoenosis is an assemblage or association of populations of
two or more different species occupying the same geographical area and in a particular
time.
IASbaba remains neutral with same answer (a)

www.iasbaba.com

80

Environment-Ecology-Biodiversity- Subject wise compilation of 60 Day Plan- 2016


Q.154) Hemlock, red pine, oak & beech are major tree species of which of the following
biome?
a) Mediterranean
b) Temperate Biome
c) Tropical evergreen biome
d) None

*Note: Question is corrected, please update the same.


Q.154) Solution (b)
Hemlock, red pine, oak & beech are major tree species of temperate biome. Found in
eastern part of USA, Great lakes region, western and central Europe, China & Japan, the
Temperate Forests otherwise called mid-latitude forest biome is one of the most altered
biomes on our planet.
Temperate forests trees are distinguished by broad leaves. They lost their leaves annually
e.g. species of oak, hickory, beech, hemlock, maple, basswood, cottonwood, elm, willow,
and spring-flowering herbs. The fauna is dominated by squirrels, rabbits, skunks, birds, deer,
mountain lion, bobcat, timber wolf, fox and bear etc.
Q.155) Identify the FYP which ushered in Blue Revolution in India
a) Fourth FYP
b) Fifth FYP
c) Sixth FYP
d) Eleventh FYP
Q.155) Solution (b)
Blue Revolution: Adoption of a package program to increase the production of fish and
marine products
Q.156) Identify the district all set to become Indias first organic Block Panchayat
a) Nellore
b) Alathur
c) Kozhikode
d) Kottayam
Q.156) Solution (b)
Organic Block Panchayat
Implementation: Hill area development Authority + Integrated watershed development
programme + Swacch Bharat Mission
Coordinating Agency: Agriculture Department
Provisions: Loans with subsidies + Supply of seeds and organic fertilizers

www.iasbaba.com

81

Environment-Ecology-Biodiversity- Subject wise compilation of 60 Day Plan- 2016

Additionally note,
Establishment of National Organic Farming Research institute: Sikkim
Q.157) Which is the nodal agency for the National Bamboo Mission?
a) Ministry of New & Renewable Energy
b) Ministry of Agriculture
c) Ministry of Environment, Forest & Climate Change
d) Federation of Green Energy
Q.157) Solution (b)
National Bamboo Mission
Launch: 2006-07
Called: An evergreen plant
3-day world conference: Indore (MP)
Q.158) Which of the following are the causes for loss of biodiversity?
1. Destruction of Habitat
2. Invasion by alien species
3. Keeping animals in Zoological parks
4. Over exploitation of natural resources
5. Climate change
Select the code from below:
a) 1,2,3 and 4
b) 1,2,4 and 5
c) 1,4 and 5
d) All of the above
Q.158) Solution (b)
Keeping animals in the Zoo helps in protecting the biodiversity.
Q.159) Consider the following statements regarding Pitcher Plant:
1. It is a carnivorous plant.
2. It does not make its food through photosynthesis because of lack of nutrients.
3. It is found in the Thar desert of India
Which of the above statement(s) are correct?
a) 1 only
b) 2 and 3
c) 1 and 3

www.iasbaba.com

82

Environment-Ecology-Biodiversity- Subject wise compilation of 60 Day Plan- 2016


d) All of the above
Q.159) Solution (a)
Pitcher plants are capable of photosynthesis but it is not enough to for their nutrient
requirement. So they gather it by ingesting other insects.
They are found in rain forests of North East India.

Q.160) Which of the following are the causes for loss of biodiversity?
1. Destruction of Habitat
2. Invasion by alien species
3. Keeping animals in Zoological parks
4. Over exploitation of natural resources
5. Climate change
Select the code from below:
a) 1,2,3 and 4
b) 1,2,4 and 5
c) 1,4 and 5
d) All of the above
Q.160) Solution (b)
Keeping animals in the Zoo helps in protecting the biodiversity.
Q.161) Consider the following statements regarding Pitcher Plant:
1. It is a carnivorous plant.
2. It does not make its food through photosynthesis because of lack of nutrients.
3. It is found in the Thar desert of India
Which of the above statement(s) are correct?
a) 1 only
b) 2 and 3
c) 1 and 3
d) All of the above
Q.161) Solution (a)
Pitcher plants are capable of photosynthesis but it is not enough to for their nutrient
requirement. So they gather it by ingesting other insects.
They are found in rain forests of North East India.

www.iasbaba.com

83

Environment-Ecology-Biodiversity- Subject wise compilation of 60 Day Plan- 2016


Q.162) Consider the following National Park/Wild life Sanctuaries and the animals they
are famous for:
National Park/ Sanctuary Specie
1. Kaziranga One horned Rhinoceros
2. Ranthambor Tiger
3. Chambal Wildlife Sanctuary Gharial
4. Kutch wildlife Sanctuary Asiatic wild Ass
Which of the above are correctly matched?
a) 1 and 2
b) 1,2 and 4
c) 2,3 and 4
d) All of the above
Q.162) Solution (d)
Self Explanatory
Q.163) Which of the following forests is known as the Lungs of the Planet Earth?
a) Tundra forest
b) Coniferous forests of Taiga
c) Amazon Rain Forest
d) Corals
Q.163) Solution (c)
Amazon Rain Forests are called the Lungs of the World because of the amount of Oxygen
they produce.
Q.164) Which of the following groups exhibit more species diversity?
a) Angiosperms
b) Algae
c) Bryophytes
d) Fungi
Q.164) Solution (a)
Angiosperms: it comprises of the plants that have flowers and produce seeds enclosed with
in a carpel.
Algae: Algae are a type of plant-like living things that can make food sunlight by
photosynthesis.
Bryophytes: They are non-vascular plants, such as mosses, hornworts and liverworts. They
do not have a true vascular tissue.

www.iasbaba.com

84

Environment-Ecology-Biodiversity- Subject wise compilation of 60 Day Plan- 2016


Fungi: A fungus is any member of the group of eukaryotic organisms that includes
unicellular microorganisms such as yeast and molds, as well as multicellular fungi that
produces familiar fruiting forms known as mushrooms.
http://www.bsienvis.nic.in/Database/Status_of_Plant_Diversity_in_India_17566.aspx

Q.165) Sounds above which decibel level are considered as physically painful?
a)
b)
c)
d)

80 dB
60 dB
30 dB
40 dB

Q.165) Solution (a)


A decibel is the standard for the measurement of noise. The zero on a decibel scale is at the
threshold of hearing, the lowest sound pressure that can be heard, on the scale acc. To
smith, 20 db is whisper, 40 db the noise in a quiet office . 60 db is normal conversation, 80
db is the level at which sound becomes physically painful.
The baseline noise levels in the community vary around 40 dB.
WHO recommends an Industrial noise limit of 75 dB.
Noise above 115dB is regarded as highly avoidable.

Q.166) Competition for light, nutrients and space is most severe between:
a) Closely related organisms growing in different Niches
b) Closely related organisms growing in same niche
c) Distantly related organisms growing in the same habitat
d) Distantly related organisms growing in different niches
Q.166) Solution (b)
Closely related organisms growing in the same niche have completion for light, nutrients
and space to maintain their population. Competition acts as a check and balance to
maintain right population and survival of healthy organisms.
Q.167) Consider the following statements:
1. Jet planes are a major contributor to Ozone depletion.
2. Jet planes fly through the stratosphere making holes in ozone layer with immense force.
Which of the above statement are correct?
a) 1 only

www.iasbaba.com

85

Environment-Ecology-Biodiversity- Subject wise compilation of 60 Day Plan- 2016


b) 2 only
c) Both 1 and 2
d) Neither 1 nor 2
Q.167) Solution (a)
Jet planes are a major contributor to ozone depletion. Main source of aerosols is the
emission of jet planes. These aerosols contains chlorofluorocarbons which react with ozone.
Q.168) Bhopal Gas tragedy is one the worst chemical tragedies of India. It was caused due
to leak of which of the following gas?
a) Methyl isocyanate
b) Uranium 235
c) Ethyl isocyanate
d) DDT
Q.168) Solution (a)
Methyl Isocyanate.
Q.169) Which of the following pyramids are upright in grassland ecosystems?
1. Pyramid of numbers
2. Energy Pyramid
Select the code from below:
a) 1 only
b) 2 only
c) Both 1 and 2
d) Neither 1 nor 2
Q.169) Solution (c)
Pyramid of numbers shows the number of individual organisms at each level. In Grassland,
the producers, which are mainly grasses, are always maximum in numbers. This number
than shows a decrease towards the apex.
Pyramid of energy is always upright for every ecosystem.
Q.170) Homeostasis means:
a) Tendency of biological systems to change with change in environment.
b) The ability to maintain steady state with changing environment.
c) Disturbance of self regulatory system and natural controls.
d) Biotic materials used in Homeopathic medicines.
Q.170) Solution (b)

www.iasbaba.com

86

Environment-Ecology-Biodiversity- Subject wise compilation of 60 Day Plan- 2016


The maintenance of a constant internal environment is called Homeostasis.
Q.171) Consider the following statements regarding Humus
1. Humus is finely divided semi-decomposed organic material.
2. It helps in binding the minerals(positive ions) in the soil and make them available to the
roots of the plants.
3. Humus is found in O and A layer of the soil.
Which of the above statements are correct?
a) 1 only
b) 1 and 3
c) 2 and 3
d) All of the above
Q.171) Solution (d)
Humus is the finely divided semi-decomposed organic material. It is negatively charged
collide and helps in binding the positive mineral ions. It is found in the top soil.
Q.172) Earlier royalty used to eat in silver wares. Which of the following metal poisoning
was sometimes caused due to that?
a) Silver poisoning
b) Lead poisoning
c) Copper poisoning
d) Arsenic poisoning
Q.172) Solution (b)
Usually Silver ore is found with lead. If it is not extracted properly, lead impurity remains in
the metal. Continuous eating in silver ware caused lead poisoning.
Q.173) Consider the following statements:
1. Texture of soil is determined by the size of the soil particles.
2. Soil texture determines the water holding capacity of the soil.
Which of the above statements are correct?
a) 1 only
b) 2 only
c) Both 1 and 2
d) Neither 1 nor 2
Q.173) Solution (c)

www.iasbaba.com

87

Environment-Ecology-Biodiversity- Subject wise compilation of 60 Day Plan- 2016


Soil texture is determined by the size of soil particles. It can be sand, silt or clay in
decreasing order of soil particles. The finer the texture, more is the water holding capacity.
Q.174) Ultraviolet radiations from sunlight cause a reaction that produces:
a) Fluorides
b) Ozone
c) Nitrogen dioxide
d) Sulphur dioxide
Q.174) Solution (b)
Ozone is generated in the stratosphere as well as lower troposphere when Oxygen
molecules react with Oxygen free radicals in presence of UV.
Q.175) Which of the below statements about Ganges River Dolphin is/are correct?
1) The Ganges River Dolphin is also known as the "Susu" or Soons is an endemic fauna of
the Ganges, Brahmaputra, and Meghna river systems.
2) They are often known as the Tiger of Indian Rivers, the river dolphin is an indicator
animal and has the same position in a river ecosystem as a tiger in a forest, its presence
indicating a sign of a healthy river ecosystem.
Select the correct code:
a) 1 only
b) 2 only
c) Both 1 and 2
d) Neither 1 nor 2
Q.175) Solution (a)
Ganges River dolphin (Status - IUCN: Endangered)
The Ganges River dolphin, or susu or soons, inhabits the Ganges-BrahmaputraMeghna and Karnaphuli-Sangu river systems of Nepal, India, and Bangladesh.
They are often known as the Tiger of the Ganges, the river dolphin is an indicator
animal and has the same position in a river ecosystem as a tiger in a forest, its presence
indicating a sign of a healthy river ecosystem.
Golden Masheer fish (Status IUCN: Endangered)

www.iasbaba.com

88

Environment-Ecology-Biodiversity- Subject wise compilation of 60 Day Plan- 2016

Golden Masheer, also known as the Tiger of Indian Rivers, is the longest-living
freshwater fish. It belongs to the genus Tor.
It is native to mountain and sub-mountain regions at altitudes of up to 2,000 metres
above sea level.
Recently, Himachal Pradesh government has announced to start an artificial propagation
programme for rehabilitation and conservation of Golden Masheer Fish. Under this
programme, a hatchery has been set up with an outlay of Rs 6 crore to rear Masheer fish
and release its fingerlings into the wild.

Q.176) Consider the following statements:


Assertion (A): The coral reefs are called as rainforests of the oceans.
Reason (R): The coral reefs are more diverse than the tropical rainforests because the coral
reefs have about 1,000,000 species
Choose the correct answer:
a) Both A and R are true and R is the correct explanation of A
b) Both A and R are true but R is not the correct explanation of A
c) A is true but R is false
d) A is false but R is true
Q.176) Solution (a)

www.iasbaba.com

89

Environment-Ecology-Biodiversity- Subject wise compilation of 60 Day Plan- 2016


Both the given statements are true.
Coral polyps commonly thrive in the tropical oceans confined between 25 N and 25 S
latitudes. They are found in the tropical oceans and seas because they require high
mean annual temperature ranging between 20 C and 21 C for their survival.
The coral reefs are more diverse than the tropical rainforests because the coral reefs
have about 1,000,000 species. This is why coral reefs are called as rainforests of the
oceans.
Q.177) Which among the following statement(s) is/are not true in regard to coral
bleaching?
1) Coral bleaching refers to loss of algae from the corals resulting in the white colour of
corals which is indicative of death of corals.
2) Siltation has been reported as the major factor of coral bleaching.
3) El Nino phenomenon also contributes to coral bleaching.
Select the correct code:
a) 1 only
b) 2 only
c) 3 only
d) None
Q.177) Solution (b)
Global warming has been reported as the major factor of coral bleaching.
Doubt- Babaji, please clarify on question No-18. According to Wikipedia, Coral bleaching
does not indicate death of corals. They can regenerate if favourable conditions occur.
Clarifications

Refer Class 10 ICSE book - Environmental Education


It says - "Coral bleaching refers to loss of algae from the corals resulting in the white colour
of corals which is indicative of death of corals"
Although corals can recover from bleaching when water temperatures drop, they can also
die if they are without their algae for too long.
http://theconversation.com/how...

Thermal stress is most visible when corals bleach. Too much heat means corals and
zooxanthellae separate, the zooxanthellae taking the colour (hence 'bleaching') and energy
sources with them. Coral bleaching is not always fatal, but has been one of the main causes
of coral death around the world in the last 20 years.

www.iasbaba.com

90

Environment-Ecology-Biodiversity- Subject wise compilation of 60 Day Plan- 2016


http://www.gbrmpa.gov.au/manag...

Statement in the ques simply says - loss of algae from the corals is serving as a sign or
indication of death of corals
So no problem with question.
Q.178) Consider the following statements in regard to Compost:
1) Greens have more nitrogen in them.
2) Nitrogen is a critical element in amino acids and proteins, and can be thought of as a
protein source for the billions of multiplying microbes.
3) A good mix of browns and greens is the best nutritional balance for the microbes,
however browns often need to be moistened before they are put into a compost
system.
Which of the above statements is/are correct?
a) 2 and 3 only
b) 3 only
c) 2 only
d) All of the above
Q.178) Solution (d)
All the given statements are correct.
Greens include fresh (and often green) plant materials such as green weeds, kitchen fruit
and vegetable scraps, green leaves and tea bags, fresh horse manure etc..
Browns include dry and dead plant materials such as straw, dry brown weeds, autumn
leaves and wood chips or sawdust etc..
Q.179) Recently, environmentalists and ecologists have given high importance to Social
forestry programme and a lot of debates focusing on Eucalyptus. Which among the
following is true in regard to it?
1) Social forestry refers to the concept of planting trees and pasture development on
village common lands, roadside and along railways, commonly in non-forest areas for
the benefit of the society.
2) Eucalyptus is grown extensively under this programme as its cultivation has proved to be
an ecological friendly.
3) This is because other plants can grow near Eucalyptus trees and they take up very less
ground water and soil nutrients.
Select the correct code:

www.iasbaba.com

91

Environment-Ecology-Biodiversity- Subject wise compilation of 60 Day Plan- 2016


a)
b)
c)
d)

1 only
2 and 3 only
1 and 3 only
All of the above

Q.179) Solution (a)


The main drawback of social forestry was
The programme has been found to benefit only the rich farmers, who can plant crops
that can be rotated in short durations and find a ready market.
Eucalyptus is grown extensively under this programme because it is a cash crop and
gives fuel wood. But eucalyptus cultivation has proved to be an ecological disaster. This
is because no other plant can grow near eucalyptus trees and they take up a lot of
ground water, so much so that the water table is lowered, soil nutrients are depleted
and the soil gets degraded in course of time.
Q.180) Helsinki Conference was associated with:
a) Conference on Environment and Cooperation in Europe, was an effort to reduce
environmental tensions between the Russia and Western blocs
b) INDCs - Intended Nationally Determined Contributions UNFCCC
c) To upend current barriers to sharing clinical data and insights on Nature Biotechnology
d) Elimination of CFC that causes ozone depletion
Q.180) Solution (d)
In May 1989, an international conference on ozone was held at Helsinki to revise the
Montreal Protocol. As many as 80 countries agreed to have a total ban by 2000 on
chemicals that cause ozone depletion. The agreement for CFC elimination is a major step
towards environmental protection.

Q.181) Consider the following statements in regard to AGMARK and FPO mark. Choose
the incorrect answer:
a) AGMARK is a certification mark employed on agricultural products in India, assuring that
they conform to a set of standards approved by the Agricultural Produce Market
Committee (APMC)
b) The AGMARK is legally enforced in India by the Agricultural Produce (Grading and
Marking) Act
c) The FPO mark is a certification mark mandatory on all processed fruit products sold in
India, following the Food Safety and Standards Act of 2006
d) The agency that develops standards for this purpose and that which issues the FPO mark
is the Ministry of Food Processing Industries of the Government of India

www.iasbaba.com

92

Environment-Ecology-Biodiversity- Subject wise compilation of 60 Day Plan- 2016


Q.181) Solution (a)
AGMARK is a certification mark employed on agricultural products in India, assuring that
they conform to a set of standards approved by the Directorate of Marketing and
Inspection, an agency of the Government of India

Q.182) Choose the incorrect answer:


a) Harike Wildlife Sanctuary - NH 15
b) Dara wildlife Sanctuary NH 12
c) Gautam Budha Wildlife Sanctuary NH 5
d) Bandipur National Park - NH-67 and NH-212
Q.182) Solution (c)
NH passing through some important Sanctuaries and National Parks (recently in news
from past 2 years)
1. The National Highway (NH209) passing through Sathyamangalam Wildlife Sanctuary
poses a grave threat to wildlife.
2. The NH 67 passes through Sathyamangalam Wildlife Sanctuary and Bandipur National
Park in Karnataka state.
3. The National Highway (NH-67) & (NH-212) passes through Bandipur national park . This
road has been a major concern as speeding vehicles have killed many wild animals in
spite of frequent warnings to travelers from the forest department officials and
restriction on movement of vehicles in some stretches between 9 P.M to 6 A.M.
4. Nauradehi Wildlife Sanctuary, MP - The Jabalpur-Jaipur highway (NH 12) passes through
this sanctuary
5. Gautam Budha Wildlife Sanctuary, Bihar NH 2 passes through this sanctuary
6. Madhav National Park, MP NH 3
7. Pench Tiger Reserve, MP - The NH 44 (old NH 7), runs between Nagpur and Jabalpur
along the eastern boundary of the reserve for around 10 km
8. Borail Wildlife Sanctuary, Assam - NH 54 (E)
9. Bhimbandh Wildlife Sanctuary, Bihar - NH 72
10. Harike Wildlife Sanctuary, Punjab - NH 15

www.iasbaba.com

93

Environment-Ecology-Biodiversity- Subject wise compilation of 60 Day Plan- 2016


11. Abohar Wildlife Sanctuary, Punjab - NH 15
12. Dara wildlife Sanctuary, Rajasthan - NH-12
13. Great Indian Bustard Sanctuary NH 13
14. Karnala Bird Sanctuary NH 17
Q.183) Consider the following differences between Crocodile and Aligator. Select the
incorrect statement.
a) Alligators have wider, U-shaped snouts, while crocodile front ends are more pointed and
V-shaped.
b) Alligators tend to live in saltwater habitats, while crocodiles hang out in freshwater
marshes and lakes.
c) In alligators, the upper jaw is wider than the lower jaw and completely overlaps it.
Therefore, when mouth closed, only the top teeth are visible.
d) In crocodiles, the upper jaw and lower jaw are approximately the same width, and so
both the top and bottom teeth are visible when mouth is closed.
Q.183) Solution (b)
Crocodiles tend to live in saltwater habitats, while alligators hang out in freshwater marshes
and lakes.

www.iasbaba.com

94

Environment-Ecology-Biodiversity- Subject wise compilation of 60 Day Plan- 2016

www.iasbaba.com

95

Environment-Ecology-Biodiversity- Subject wise compilation of 60 Day Plan- 2016


Q.184) The Red list of threatened species is prepared by the:
a) Convention on International Trade in Endangered Species of Wild Fauna and Flora
(CITES)
b) International Union for Conservation of Nature (IUCN)
c) UN Environment Assembly (UNEA)
d) UN Environment Programme (UNEP)
Q.184) Solution: (b)
International Union for Conservation of Nature (IUCN)
Q.185) Consider the following statements:
1. Genetic diversity refers to the variation of genes within species.
2. Specie richness refers to the variety of species within a region.
Select the incorrect statement:
a) Only 1
b) Only 2
c) Both 1 and 2
d) Neither 1 nor 2
Q.185) Solution: (b)
Species diversity refers to the variety of species within a region.
Specie richness (simplest measure of species diversity) refers to the number of species
per unit area.
Q.186) Which of the following has been set up under MAB program of UNESCO?
a) National Park
b) Sanctuary
c) Buffer Zone
d) Biosphere Reserves
Q.186) Solution: (d)
Q.187) Where is Kedarnath Sanctuary located?
a) Bihar
b) Himachal Pradesh
c) Uttarakhand
d) Uttar Pradesh
Q.187) Solution: (c)
Uttarakhand (Musk Dur Project Himalayan musk deer)

www.iasbaba.com

96

Environment-Ecology-Biodiversity- Subject wise compilation of 60 Day Plan- 2016

Q.188) Identify the State:


1. Lai Haroba, Shim Lam and Thang-Ta are the trademark dances of this State.
2. The sacred groves are known as Umanglai.
3. Sangai is the State Animal and Nongin, its State Bird.
Select the correct option:
a) Mizoram
b) Manipur
c) Nagaland
d) Assam
Q.188) Solution: (b)
Q.189) What do you mean by Kanams?
a) Backwaters of Kerala
b) Patch of biodiversity-rich small groves
c) Coral reef off the coast of Tamil Nadu
d) A small island near Lakshwadeep
Q.189) Solution: (b)
http://www.thehindu.com/sci-tech/a-green-world-lies-hidden-in-keralas-lateritehills/article8085568.ece
Q.190) Identify this animal with CITES listing
1. Its fur is famously known as Shahtoosh which is used to make luxury shawls.
2. It is listed as endangered.
Select the correct animal:
a) Lion-tailed Macquae
b) Slow Loris
c) Blue-eyed Black Lemur
d) Tibatian Antelope
Q.190) Solution: (d)
Q.191) BIOFIN is
a) A portable remote imaging spectrometer
b) Hybrid of debt & equity financing model for addressing the lack of finance in leveraging
Indias efforts to safeguard its environment
c) A pilot project to measure the hearing sensitivity of whales

www.iasbaba.com

97

Environment-Ecology-Biodiversity- Subject wise compilation of 60 Day Plan- 2016


d) A global partnership to help government plan and pay for its actions on biodiversity
conservation and sustainable use
Q.191) Solution: (d)
Q.192) Consider the following statements regarding Peatlands
1. All wetlands include peat
2. Peatlands act as huge source of carbon
3. Peatlands regulate regional and local climate
Select the correct statement/s
a) Only 1
b) 1 and 2
c) 1, 2 and 3
d) 2 and 3
Q.192) Solution (d)
Peatlands occur in a great variety of forms and are present in more than half of all Ramsar
sites throughout the world. Unwise use of this key wetland type may damage the features
that make peatlands so important for long-term carbon storage. Drainage of peatlands also
leads to loss of carbon and fertile soil, including severe soil subsidence and salt water
intrusion in lowland coastal areas.
Peatlands
Pristine peatlands are characterized by the presence of water and special vegetation. The
peat soil, often exceeding many meters in depth, consists of organic material and water and
is created by the accumulation of partially decomposed plant materials. The layers of peat
build up over sometimes thousands of years and preserve other materials including pollen
grains, human artefacts and ancient bodies, giving us an unrivalled window into the past.
Global carbon store and climate mitigation
It has been estimated that peatlands contain at least 550 Gt of carbon, which is almost
double the amount stored in the worlds forests. However, when peatlands are drained
huge amounts of carbon dioxide (CO2) are released making restoration of great importance
for climate change mitigation
Regulation functions of Peatlands:
regulation of global climate
regulation of regional and local climates
regulation of catchment hydrology
regulation of catchment hydrochemistry
regulation of soil conditions
About 60% of the world's wetlands are peat.
http://www.peatsociety.org/peatlands-and-peat/functions-peatlands

www.iasbaba.com

98

Environment-Ecology-Biodiversity- Subject wise compilation of 60 Day Plan- 2016


http://www.ramsar.org/sites/default/files/documents/library/fs_8_peatlands_en_v5.pdf
Doubt: Please clarify babaji. Peat-land is one of the wetlands and act as carbon sink and
plays a major role in carbon sequestration but if it is degraded then only it will be acting as
carbon source. Please clarify babaji.
Clarification:
There was ambiguity in the second statement which should read like
Q.) Consider the following statements regarding Peatlands
1. All wetlands include peat
2. Peatlands act as huge sink of carbon
3. Peatlands regulate regional and local climate

Q.193) The definition of the word wetland, under the Convention, is any land area that is
saturated or flooded with water, either seasonally or permanently. Which of the given
options form part of an Inland wetland?
1. Flood Plains
2. Swamps
3. Seagrass Meadows
4. Aquifers
5. Estuaries
6. Marshes
7. Coral Reefs
Select the correct option
a) 1, 2, 3, 6 and 7
b) 2, 3, 4, 5 and 6
c) 1, 2, 4 and 6
d) 1, 2 and 5
Q.193) Solution (c)
The definition of the word wetland, under the Convention, is any land area that is
saturated or flooded with water, either seasonally or permanently. Inland wetlands include
aquifers, lakes, rivers, streams, marshes, peatlands, ponds, flood plains and swamps.
Coastal wetlands include all coastlines, mangroves, saltmarshes, estuaries, lagoons,
seagrass meadows and coral reefs.
Q.194) Consider the following regarding Conservation International
1. It is an international organization under United Nation Environment Program (UNEP)
that aims to help stabilize global climate, protect fresh water, and ensure human wellbeing

www.iasbaba.com

99

Environment-Ecology-Biodiversity- Subject wise compilation of 60 Day Plan- 2016


2. Biodiversity Hotspots is an initiative of Conservation International
Select the incorrect statement/s
a) Only 1
b) Only 2
c) Both
d) None
Q.194) Solution (a)
Conservation International (CI)
An American non-profit environmental organization headquartered in Arlington, Virginia. Its
goal is to protect nature as a source of food, fresh water, livelihoods and a stable climate.
It employs scientists and policy experts to balance healthy ecosystems with sustainable
human use. Conservation International aims to help stabilize global climate, protect fresh
water, and ensure human well-being. To achieve their goals they work with indigenous
peoples and non-governmental organization. Conservation International's primary initiatives
include climate, fresh water, food, health, culture, and biodiversity.
Of all the significant initiatives Conservation International has achieved, its Biodiversity
Hotspots project is the most impressive. This project identifies and protects biological
hotspotsplaces that exhibit the richest diversity and most threatened collections of plants
and animals on our planet.
Q.195) Identify the correct statement regarding Therukoothu
a) A Martial Art form of Tamil Nadu
b) A stage drama prevalent in Andhra Pradesh
c) A traditional folk art form of Tamil Nadu.
d) A form of regional folk dance of Karnataka
Q.195) Solution (c)
http://www.thehindu.com/features/friday-review/theatre/from-the-village-ofkoothu/article6962630.ece
http://www.livemint.com/Leisure/Zg0RExEogup782Wn0V90zH/The-Mint-Planner-19February-2016.html
Q.196) Consider the statements w.r.t Sacred Groves
1. It is included in Wildlife Protection Act, 1972
2. Nakshatravana, is a sacred grove consisting of trees
3. The sacred groves are also found in Rajasthan
Select the correct statement/s
a) 1 and 2

www.iasbaba.com

100

Environment-Ecology-Biodiversity- Subject wise compilation of 60 Day Plan- 2016


b) 2 and 3
c) Only 1
d) 1, 2 and 3
Q.196) Solution (b)
Historical references to sacred groves can be obtained from ancient classics as far back as
Kalidasa's Vikramuurvashiiya. There has been a growing interest in creating green patches
such as Nakshatravana.
Sacred groves did not enjoy protection via federal legislation in India. Some NGOs work with
local villagers to protect such groves. Traditionally, and in some cases even today, members
of the community take turns to protect the grove. However, the introduction of the
protected area category community reserves under the Wild Life (Protection) Amendment
Act, 2002 has introduced legislation for providing government protection to community
held lands, which could include sacred groves.
Doubt: Babaji; In wildlife protection act; 4 protected areas namely 1. national park 2.
wildlife sanctuaries 3. conservation reserves 4. community reserves Sacred groves fall under
the community reserves and so it comes under the wildlife protection act. please clarify
babaji;
Clarification: First statement is ambiguous. It should read as Sacred Grooves were
introduced through Wildlife Protection Act, 1972 changing the question to
Q.) Consider the statements w.r.t Sacred Groves
1) Sacred Grooves were introduced through Wildlife Protection Act, 1972
2) Nakshatravana, is a sacred grove consisting of trees
3) The sacred groves are also found in Rajasthan

Q.197) Mawphlang sacred forest is found in which of the following north-eastern state of
India
a) Sikkim
b) Manipur
c) Meghalaya
d) Nagaland
Q.197) Solution (c)

www.iasbaba.com

101

Você também pode gostar